Orthopedic NP Review Course

¡Supera tus tareas y exámenes ahora con Quizwiz!

Teardrop vertebral fracture

compression and flexion that fractures the anterior portion of the vertebrae, can injure spinal cord

Risk factors for DDH

first born; breech presentation; female; family hx

Spiral fracture

ragged break occurs when excessive twisting forces are applied to a bone

Delayed union

Delayed union is a continuation of or increase in bone pain and tenderness beyond a reasonable healing period; healing is slowed by not completely stopped.

A patient with chronic gout is being seen by a primary care NP, who calls the orthopaedic NP for consultation about the patient's x-rays. The orthopaedic NP should identify which of the following x-ray findings as consistent with chronic gout? Mosaic pattern Punched-out lesion Osteolytic lesion Sclerotic changes

Punched-out lesion. In chronic gout, x-rays will show "punched out" erosion of bone with overhanging bony margins. A mosaic pattern on x-rays suggests Paget's disease. Osteolytic changes (softening of bone) may be related to bone tumors or other conditions. Sclerotic changes (thickening of bone) may be seen in conditions such as ankylosing spondylitis.

A 30-year-old male is seen in the office after recent diagnosis with reactive arthritis. Which of the following statements by the patient should demonstrate to the NP the patient's accurate understanding of the course of this disease? "I have about a 50% chance of developing chronic arthritis." "I have about a 75% chance of redeveloping symptoms." "Arthritis symptoms respond well to antibiotic treatment." "No treatment has a proven long-term effect."

"No treatment has a proven long-term effect." No treatment regimen has been shown to have a lasting effect on the disease course. Approximately 20% of patients with reactive arthritis develop chronic arthritis, while 15%-50% redevelop symptoms after the initial flare has ended. Antibiotics are used to treat confirmed infection; they have no effect on arthritis or other symptoms.

A nurse from the medical-surgical unit asks the NP about use of radiation therapy for primary sarcoma. Which of the following should be the NP's response? "Radiation therapy increases overall survival." "Radiation therapy decreases the chance of local recurrence." "Radiation therapy is not used as adjuvant therapy." "Radiation therapy decreases the chance of secondary cancer."

"Radiation therapy decreases the chance of local recurrence." Radiation therapy (RT) has been shown to decrease the chance of local recurrence in some sarcomas that are radioreceptive. The use of radiation has not been shown to increase overall survival significantly; however, retrospective studies show an improved overall survival rate with local control of the sarcoma. RT is used often as an adjuvant therapy for soft tissue tumors. Secondary cancers may occur years later as a result of DNA changes caused by the radiation.

A patient with rheumatoid arthritis recently was prescribed methotrexate 20 mg weekly. During an office visit with the NP, the patient requests additional information on this medication. Which of the following statements should the NP accurately make regarding methotrexate? "GI upset can be reduced by taking the med in the am." "Interstitial pneumonitis can occur frequently while taking this med." "Extraocular muscle movement can occur, requiring an annual eye exam." "Stomatitis can be improved w daily use of a folic acid supplement."

"Stomatitis can be improved w daily use of a folic acid supplement." Stomatitis, oral ulcers, and mild alopecia can be improved with the daily intake of 1 mg of folic acid. Extraocular muscle movement is associated with the use of hydrochloroquine (Plaquenil). Hepatic cirrhosis, interstitial pneumonitis, and severe myelosuppression occur only rarely. Gastrointestinal upset is reduced by taking the medication at night.

Sublaxation

'Dead arm syndrome' - numbness and tingling. Mechanism: forced abduction w external rotation. Tx: remove from activity and ice. Return to practice/game: full ROM, full strength, all pain free. Sidelined: 1-3 weeks.

Common findings of hyperparathyrodism

'STONES AND BONES' are the most common findings (bone disease - weak bones, and renal stone disease/nephrolithiasis) - these are accepted as universal to hyperparathyroidism. Symptoms are not necessarily correlated with serum Ca++ level, but more commonly to how rapidly the serum Ca++ rises.

Talipes Equinovarus

(clubfoot) congenital deformity of the foot in which it is plantar flexed and inverted. Calf smaller, Achilles shorter. Treatment: stretching, casting, surgical release.

Finger sprain treatment

*Splint in position of function* and refer to orthopedics; can "buddy tape" to adjacent finger. Refer for xray. Collateral sprains: splint until pain resolves (PIP 30-40 d flexion; DIP full extension), buddy tape for activity. Volar sprains: splint 20-30 d flexion x 3 wks, buddy taping.

Apley's Scratch Test

- Used to evaluate ROM in the shoulder - Test abduction/external rotation ask patient to reach behind the head and touch the opposite shoulder - To evaluate internal rotation and adduction, ask pt to reach in front of the head and touch opposite shoulder - To evaluate further internal rotation and adduction, instruct the patient to reach behind the back and touch the inferior angle of the opposite scapula. Diagnosis suggested by positive result: loss of ROM: rotator cuff problem

Ottowa Knee Rules

- any of these criteria = knee xray - 55 and older -isolated tenderness of the patella - tenderness at the head of the fibula - inability to flex to 90 degrees - inability to bear weight both immediately and in the ER (4 steps; unable to transfer weight twice onto each lower limb regardless of limping)

Septic Joint - Nongonococcal

-80-90% monoarticular -Most develop from hematogenous spread -Most common: Gram positive aerobes (80%); majority with Staph aureus (60%); gram negative 18%

Gout

-Caused by monosodium urate crystals -Most common type of inflammatory monoarthritis -Typically: first MTP joint, ankle, midfoot, knee -Pain very severe; cannot stand bed sheet -May be with fever and mimic infection -The cutaneous erythema may extend beyond the joint and resemble bacterial cellulitis

Gout treatment goals

-Gout can be treated without complications -Therapeutic goals include: terminating attacks; providing control of pain and inflammation; preventing future attacks; preventing complications such as renal stones, tophi, and destructive arthropathy

Colchicine for acute gout

-Inhibits microtubule aggregation which disrupts chemotaxis and phagocytosis -Inhibits crystal-induced production of chemotatic factors -Administered orally in hourly doses of 0.5 to 0.6 mg until pain and inflammation have resolved or until GI side effects (diarrhea) prevent further use. Max dose 6mg/24hr -2mg IV then 0.5mg q6hr until cumulative dose of 4mg over 24hr

Synovial fluid analysis

-Less viscous seen with inflammation -Cloudy - infection, WBC, Crystals -Reddish = blood -Glucose - significantly lower w/infection and inflammation -Protein increased with infection -LDH - increased with infection, RA, gout -Uric acid = gout

Tests to perform on synovial fluid

-Low threshold for doing Gram stain and cultures. -Total leukocyte count/differential: inflammatory vs. non-inflammatory. -Polarized microscopy to look for crystals

Septic Joint

-Most articular infections - a single joint -15-20% cases polyarticular -Most common sites: knee, hip, shoulder -20% patients afebrile -Joint pain is moderate to severe -Joints visibly swollen, warm, often red -Comorbidities: RA, DM, SLE, cancer, etc

Septic Joint: Gonococcal

-Most common cause of septic arthritis -Often preceded by disseminated gonococcemia -Sexually active individual, 5-7 days h/o fever, chills, skin lesions, migratory arthralgias and tenosynovitis -> persistent monoarthritis -Women often menstruating or pregnant -Genitourinary disease often asymptomatic

Acute gout attack treatment

-NSAIDS most commonly used, all work the same, Indocin most commonly used, remember to use with caution with CAD, GI bleed, RF -Indocin 50 mg PO BID-TID for 2-3 days and then taper -Ibuprofen 400mg PO q4-6hr max 3.2g/day -Ketorolac 60 mg IM or 30mg IV x1 dose in pts <65 (30mg IM or 15mg IV in single dose in pts >65 yo, or w pts who are renally impaired -Continue meds until pain and inflammation have resolved for 48 hr

Risk factors for gout

-Primary gout: obesity, HLD, DM, HTN, atherosclerosis -Secondary gout: alcoholism, drug therapy (diuretics, cytotoxics), myeloproliferative disorders, chronic renal failure

Gout presenting s/s

-Systemic: fever rare but may occur, chills and malaise -MS: Acute onset of monoarticular joint pain. First MTP most common. Usually affected in 90% of patients with gout. Other joints include knees, foot and ankle. Less common in upper extremities (Postulated that decreased solubility of MSU at lower temps of peripheral structures such as toe and ear) -Skin: warmth, erythema and tenseness of skin overlaying joint. May have pruritis and desquamation. -GU: Renal colic with renal calculi formation in patients with hyperuricemia

New therapies for gout management

-Uricase (enzyme that oxidized uric acid to a more soluble form. Natural Uricase from Aspergillus flavus and Candida utilis under investigation) -Febuxostat (new class of Xanthine Oxidase inhibitor. More selective than allopurinol. Little dependence on renal excretion) -Losartan (ARB given as 50mg/dL can be urisuric. When given with HCTZ, it can blunt the effect of the diuretic and potentiate its antihypertensive action -Fenofibrate (studies note when used in combo with Allopurinol produced additional lowering of the urate)

Arthrogryposis Multiplex Congenita (AMC)

-non-progressive neuromuscular disorder -*multiple congenital contractures at birth* -cylinder-like extremities, significant and multiple contractures, dislocation of joints, muscle atrophy -limitation of ROM & strength=> preclude reaching developmental milestone. Contractures are d/t fibrosis of affected muscles. The cause is unknown. Incidence is 0.03% of population.

Infancy

0-18 months old

Treatment of DDH

0-6 months old: triple diapers; Pavlik harness if indicated: dislocatable hips, pt under 6 mo, femoral head points to triadiate cartilage w flexion, abduction. 6-12 mo (or if Pavlik harness fails): traction (Bryant's or other balanced skin traction); closed reduction; +/- adductor tenotomy; +/- arthrogram; double hip spica cast for 3-4.5 months; open reduction if closed reduction fails. >18 mo: anterior open reduction; femoral shortening (alternatives to preliminary traction); +/- acetabular procedure (Salter, Pemberton).

Stages of Lyme disese

1st: erythema migrans (bulls eye) rash 7-10 days; headache/fever/malaise; treated w antibiotics. 2nd: days to mo later, early disseminated w a hematogenous spread (spread in the blood). Later stage: If untreated 10% develop chronic arthritis; bells palsy/myopericarditis/poor memory

Functional Capacity Evaluation (FCE)

1-4 metabolic equivalents (METs) include eating, dressing, using the bathroom, walking indoors, walking 1-2 blocks on level ground, and doing light housework. 4-10 METs include climbing stairs or walking up a hill; running a short distance; scrubbing floors or moving furniture; and recreational activities, such as golf, bowling, dancing, and playing tennis.

Adolescence

13-19 years old

Toddler

18 months - 3 years old

Prognosis of RA

2 divergent course: 50-75% experience remission in 2 years (these pts are negative for rheumatoid factor and have goof functional status even during disease activity). Conservative therapy is advised for this group. Pt who have severe disease have a worse prognosis, and on an average die 10-15 yrs earlier than people e/o RA. Since most of the joint damage occurs in the first 2 yrs, these patients should be started on a disease modifying agent early

Neurofibroma

2nd & 3rd decades of life; M=F; any nerve, 10% association w Von Recklinghausen Disease (a genetic disorder characterized by the growth of tumors on the nerves. The disease can also affect the skin and cause bone deformities), 90% are solitary; slowly enlarging masses; Café Au Lait spots; Elephantitis Neuromatosa (This lesion, in its most extreme form, may involve an entire extremity, with gigantic hypertrophy of the skin, soft tissues and the underlying skeleton)

Intoeing

3 types - metatarsus adductus, tibial torsion, femoral anteversion. Often self corrects.

Femoral Acetabular Impingement

3 types: CAM lesion bump on femoral head causing irritation to hip articular cartilage and pain; PINCER lesion otesophytic outgrowth of acetabulum causing irritation to the articular cartilage; combined lesions. Symptoms include pain in groin area or lateral hip pain w sharp stabbing pain when turning, twisting or squatting. Check for hip impingement by flexing knee and internal rotation. Dx: hip xrays AP & Dunn view; MR arthrogram of hip for labral tears. Tx: modify activities; NSAIDs; PT. Surgery: hip arthroscopy to repair labrum or clean & shave off lesions CAM and PINCER. Sometimes hip arthrotomy is needed.

Pre-school

3-5 years old

Liposarcoma

4th - 6th decade of life; high rate of metastasis; rapid growth; often painless

School age

6-12 years old

The majority of osteosarcomas occur in which region?

60% occur in the region of the knee joint, followed by hip and pelvic regions, which are 4x less likely to develop an osteosarcoma.

Anatomy of the spine

7 cervical vertebra 12 thoracic 5 lumbar 5 sacral fused vertebra (Sacrum and Coccyx). THe vertebral column i sthe part of the axial skeleton that surrounds and protects the spinal cord, while bearing the weight of the head, neck, and trunk. The vertebral column is not straight; its curves accomodate the thoracic and abdominopelvic viscera and also balance the weight of the trunk and head over the lower limbs. Cervical is concave on the posterior surface, thoracic is convex on the posterior surface, lumbar is concave, and the sacrum is convex. T4 = nipple line T10 = umbilicus L4 = iliac crest The adult human vertebral column consists of 26 bones total. Generally, 1 spinal nerve emerges at each vertebrae, save in the cervical spine where there are 7 vertebrae but 8 cervical nerves.

Cervical spine

7 vertebrae 8 cervical nerve root Atlas C1 Axis C2 has the odontoid process which fits inside C1. The 1st cervical vertebrae is called the atlas; its superior articular processes have facets that articulate with the occipital condyles of the skull in a type of joint that permits forward-backward motion of the head. The body of the 2nd cervical vertebrae, the axis, has a prominent odontoid process that extends superiorly and articulates with a facet on the atlas, providing a pivot point to allow rotational movement of the head. The facet of the superior articular process of the axis articulates with a similar flat surface on an inferior articular process of the atlas. Like other individual vertebrae, the axis has a prominent dorsal spinous process, which is notched as it is in cervical vertebrae 3-6 and is referred to as bifid; the atlas has a smaller dorsal process known as the posterior tubercle. Laterally, a transverse process provides attachment sites for muscle, while the transverse foramen allows passage of vertebral arteries and veins.

Primary trauma assessment

A - airway B - breathing C - circulation D - Da limb, da brain E - environmental controls - keep injured warm; Exposure of injured area to assess damage - only if it will not cause more harm

Torus fracture

A deformity in children caused by the longitudinal compression of the soft bone in either the radius or ulna, or both, and characterized by localized bulging

Cerebral palsy

A loss or deficiency of motor control with involuntary spasms caused by permanent brain damage present at birth. Non-progressive.

Correlation

A measure of the extent to which two factors vary together, and thus of how well either factor predicts the other.

Blunt force trauma

A non-penetrating injury that results from force applied to the body. Compression: impact/shorten. Shearing: tearing/dissection. Torsion/twisting: dislocation/sublaxation. Tensile/Traction: stretching/tearing.

Making the diagnosis: guidelines for osteoporosis for 50 yo +

A patient is diagnosed with osteoporosis if they meet any one of the following criteria: * T-score -1.0 to -.25 & 10-yr probability of hip fx >/= 3% or any fx risk >/= 20%. * T-score < -2.5 at femoral neck, total hip or spine after appropriate eval to exclude secondary causes. * T-score -1.0 to -2.5 secondary causes associated w/high fx risk. * A hip or vertebral fragility fx. * Other prior fragility fx & T-score -1.0 to -2.5.

Analysis of Variance (ANOVA)

A procedure used to test equality of three or more means.

Jersey Finger

A rupture of the flexor digitorum profundus (FDP) tendon from the distal phalanx because of the rapid extension of the finger while actively flexed. Presents with: inability to flex finger tip. Initial tx: splint in position; repair w/in 7 days. Complications if missed: retraction into palm loss of flexion of tip; impaired work ability; difficult surgery.

Second impact syndrome (SIS)

A second injury to the brain during the vulnerable metabolic cascade. May be a minor/incidental injury. Can lead to severe worsening of mental status and even death.

The NP is discussing osteoporosis treatment options for a patient who sustained a low energy fracture of the proximal humerus. Given the patient's history of venous thromboembolism, which of the following should the NP recognize as contraindicated for this patient?

A selective estrogen receptor modulator (SERM) such as raloxifene can cause a blood clot or stroke and is contraindicated for any patient with history of VTE.

T-test

A test used to determine if the means of 2 sets of data are statistically different from each other

External fixation

A versatile method of immobilization that employs percutaneous transfixing pins/wires in bone attached to a rigid external frame. Allows wide range of anatomic correction both congenital and acquired. Types: circular frame, semicircular frame, unilateral frame, bilateral, quadrilateral frame. Pin care: 50% hydrogen peroxide and water daily with dressing change.

Basic principles of trauma treatment

ABCs Identify life threatening injuries: head/neck/spine; chest; pelvis/femur/open long bone fx. Splint the injury: Splint at it lies: cervical spine; immobilization w hands/cervical collar; pelvis/femur: backboard/traction splint/external fixation.

Medical conditions associated w bone loss

AIDS/HIV; ankylosing spondylitis aka Bechterew disease; blood & bone marrow disorders; breast CA; Cushing's syndrome; eating disorders; emphysema; female athlete triad; gastrectomy; RA; liver disease; spinal cord injury; stroke; thalassemia; thyrotoxicosis; weight loss; hyperparathyroidism; hyperthyroidism; idiopathic scoliosis; inflammatory bowel disease; DM; kidney disease; lupus; lymphoma & leukemia; celiac disease/Crohns; multiple myeloma; organ transplants; Parkinson's disease; poor diet; post-polio syndrome premature menopause; prostate cancer.

Complex Regional Pain Syndrome (CRPS)

AKA Reflex Sympathetic Dystrophy Cause is unknown. Usually begins about 1 month after injury. Frequently occurs after a minor injury. Pain out of proportion, edema, skin moistness, joint stiffness, vasomotor instability, skin/hair changes. Diagnosed clinically, bone scan may show increased activity. Treatment is difficult, may try pain meds, PT, nerve block, TENS, pain specialist.

Systemic Sclerosis

AKA scleroderma Etiology: autoimmune disorder; multi-system (microvascular and connective tissue). Vasculopathy and fibrosis. Alterations in skin and internal organs. Limited (CREST syndrome) and diffuse types. Incidence: est 100,000 in US. Usually age 30-50. Women > men. All races and ethnic groups. Pulmonary fibrosis is the leading cause of death for this disease.

Pelvic ring fracture

AP compression: classic open book, anterior disruption and widening at the symphysis pubis and posterior disruption at the ilium or sacroiliac joint. Lateral compression: hit by car from side, rami fractures, iliac and sacroiliac joint fractures. Vertical shear: fall from height, hemipelvis may be displaced vertically, pubic rami and sacroilliac fracture. Complex: 2 or more of the above fractures.

ASA Physical Status Classification System

ASA 1 Normal healthy patient ASA 2 Patients with mild systemic disease ASA 3 Patients with sever systemic disease ASA 4 Patients with severe systemic disease that is a constant threat to life ASA 5 Moribund patients who are not expected to survive w/o the operation ASA 6 A declared brain-dead patient whos organs are being removed for donor purposes

Treatment of NSAID gastropathy

Acute bleed or perforation: Stop NSAID, endoscopy or surgery, start omeprazole. Ulcer w/o bleed or perforation, and needs or wants continued NSAID: omeprazole 20 mg qd - 76% healed; OR misoprostol 200 uq qid - 71% healed

Hypoparathyroidism management

Acute clinical management: acute condition is a medical emergency (i.e. after thyroidectomy postop phase): prevent laryngeal spasms, watch for tetany, seizures, give IV Ca++, gluconate/chloride STAT, ensure Mag is normal. Long term clinical management: lifelong Vit D therapy; lifelong Ca++ supplementation: 1-3 g/d; muscle relaxants prn; rx to decrease GI absorption of phosphorus (Renvela); treat ETOH abuse if present.

Indications for external fixation

Acute trauma to bones w/w/o segmental loss; limb length discrepancies; infected and non-infected long bone union and nonunion; osteomyelitis; angulation and soft tissue deformities; correction of chronic or residual deformities.

Fibromyalgia treatment

Aerobic exercise; Cog behavioral tx; pt education; strength training; acupuncture; biofeedback; hypnotherapy; antidepressants; analgesics; anticonvulsants (pregabalin - Lyrica); SNRI (duloxitine - Cymbalta)

Achilles Tear

Age 30-40. 75% during athletic activity. Episodic athlete or weekend warrior. Hear or feel a pop + pain. Weakness in push-off or unable to walk. Thompson Test - planter flexion of the foot is lost. Inability to perform a single heel rise (may be 4/5 plantarflexion strength). Palpable defect. Risk factors: poorly vascularized area 2-6 cm proximal to insertion; fluoroquinolones (Cipro); DM; contralateral rupture (>100 x increased risk); serum lipid profile; extracorporeal shock wave therapy; immobilization; age; steroids (local or systemic)

A patient presents to the office with a new diagnosis of osteoporosis. In reviewing the history, the NP notes the patient has taken prednisone 10 mg daily for asthma for 20 years. Which of the following medications should the NP recognize as the MOST appropriate treatment for possible glucocorticoid-induced osteoporosis? Alendronate (Fosamax) Risedronate (Actonel) Raloxifene (Evista) Teriparatide (Forteo)

Aldendronate, risedronate, raloxifene, and teriparatide are all prescribed to treat osteoporosis. However, only RISENDRONATE is indicated for the treatment and prevention of glucocorticoid-induced osteoporosis in men and women who use 7.5 mg or more of prednisone for chronic treatment.

Laboratory studies in the investigation of tumors/cancer

Alk Phos - increased in Paget's & Osteogenic Sarcoma; SPEP & UPEP for Multiple Myeloma; CRP, ESR, CBC w diff for infectious disease vs neoplastic

Assess for osteoporosis if:

All women beyond menopause need to be assessed for multiple risk factors related to osteoporosis as well as BMD; Men and women over 50 who have suffered a fragility fracture should be evaluated for bone loss; corticosteroid use at any time; unexplained back pain; weight loss; don't miss Vit D deficiency; use FRAX to determine need to treat.

Metabolic bone disorders

An umbrella term, disorders of bone strength and quality usually caused by abnormalities of minerals (such as Ca+, Phos, Vit D) and hormones. These disorders affect bone mass &/or structure. Include: hyper and hypoparathyroidism, osteomalacia, Paget's disease, osteoporosis.

Experimental design study

Always has 3 things: manipulation of an independent variable; control over experiment (such as a control group); and random assignment to group.

Use of an anteriogram for a tumor

An anteriogram provides information about the tumor's blood supply and maps a route for delivery of intra-arterial chemotherapy. Tumor necrosis can also be measured to determine an appropriate date for surgery. The overall goal is to shrink the tumor for surgical removal.

Malignant hyperthermia

An inherited disorder of skeletal muscle triggered in humans or animals in most stances by inhalation agents and/or succinylcholine, resulting in hypermetabolism, skeletal muscle damage, hyperthermia, and death if untreated. Underlying physiologic mechanism - abnormal handling of intracellular Ca+ levels. Triggering agents: potent volatile anesthetics (e.g. Halothan, Sevoflurane, desflurane). Non-triggering: IV agents, opioids, non-depolarizing agents succinylcholine, ketamine, propofol, anxiolytics

Osgood-Schlatter's Disease

An irritation of the patellar ligament at the tibial tuberosity. Incomplete separation of tibial tuberosity. M>F. F: 8-13 yo. M: 11-15 yo. Sx: painful swelling at the tibial tubercle. Cosmetic deformity. Dx: exam; xray traction apophysitis. Tx: rest, ice, stretch hamstrings, casting if needed.

A patient has a low N-telopeptide with a new osteoporotic fracture. What medication should they be placed on?

Anabolic agent. Cross-linked NTx measures bone collagen breakdown products, with a high value indicating high turnover and a low value indicating low turnover. Only anabolic agents (e.g., parathyroid hormone) are recommended for patients with low NTx. The other medications are prescribed for patients with high NTx. The clinical use of iPTH and teriparatide action is based on its effect of stimulating bone formation before it enhances bone resorption, the period when they are maximally anabolic (anabolic window).

Periarticular inflammation

Anatomic structure: Tendon, bursa, ligament, muscle, bone (outside the joint) Painful site: focal "point" Pain on movement: Active, in a few planes Swelling: uncommon

Articular inflammation

Anatomic structure: synovium, cartilage, capsule (whithin the joint) Painful site: diffuse, deep Pain on movement: Active/passive, all planes Swelling: common

Special test for knee injury complaint

Anterior drawer/Lachman - ACL Posterior drawer/posterior sag - PCL Valgus stress - MCL Varus stress - LCL Apprehension test - patellar instability McMurray's/Apley's/Thessaly tests - menisci

The NP is evaluating a patient in the office and suspects the patient may have rheumatoid arthritis. The NP should order which of the following tests because of its high specificity for diagnosis of rheumatoid arthritis? Anti-nuclear antibodies (ANA) Antibodies against citrullinated proteins (Anti-CCP) WBC Anti-mitochondrial antibody (AMA)

Antibodies against citrullinated proteins (Anti-CCP) Antibodies against citrullinated proteins (anti-CCP) are highly specific for diagnosis of RA; more than 80% of patients with RA have anti-CCP antibodies. ANA is positive in only 20%-30% of patients with RA and is not considered diagnostic. WBC would be normal in RA. AMA is not indicated for RA workup.

Osteoporosis treatment options

Antiresoprtive agents: bisphosphonates; selective estrogen receptor modulator; rank ligand inhibitor (dunosumab). Anabolic agents: synthetic parathyroid hormone (PTH teriparatide) - stimulates osteoblastic activity preferentially to osteoclastic activity; rebuilds bone by this mechanism.

A patient is evaluated by the NP following treatment for a fragility fracture. A DEXA scan is done and the patient is diagnosed with osteopenia. When prescribing alendronate (Fosamax) for this patient, the NP should explain this medication:

As a bisphosphonate, alendronate impairs the function of the osteoclasts.

What is the first nursing priority after spica cast application?

Assessment for swelling or cast syndrome is the priority

Malignant hyperthermia (MH) prevention

Avoid MH trigger agents in MH susceptible or those suspected of being susceptible. Preop personal/family hx of anesthetic problems, neuromuscular disorders to identify those who may be MH-susceptible. Temp/end tidal CO2 monitoring during general anesthesia. Recognition of masseter muscle rigidity. Prompt investigation of unexplained tachycardia, herpcarbia, hyperthermia. Availability of Dantrolene. ORs should perform regular MH drills to be prepared.

Prevention of NSAID gastropathy

Avoid the problem: stop the NSAID and use alternative treatment: low-dose prednisone, Tylenol, Nonacetylated salicylates. Use a selective cyclooxygenase-2 inhibitor. Antacids and H2 blockers are not the answer! May mask symptoms but DO NOT prevent serious events

Gamekeeper's thumb

Avulsion of thumb ulnar collateral ligament (UCL) at the ulnar aspect of first metacarpophalangeal (MCP) joint. Acute or chronic. Common injury w skiers, snowboards, jet skiers, etc. Pain at ulnar border of thumb - metacarpal phalangeal (MCP) joint.

Intra-articular injection with steroids for acute gout

Beneficial in pt with 1-2 large joints affected. Good option for elderly pt with renal or PUD (peptic ulcer disease) or other illness. Triamcinolone 10-40mg or Dexamethasone 2-10mg alone or in combination with Lidocaine

DMARDS

Biologics: Non-TNF (Abatacept, Rituximab, Tocilizumab, Tofacitinib, Anakinra) & Anti-TNF (Adalimumab, Certolizumab, Etanercept, Golimumab, Infliximab) Non-Biologics: Hydroxychoroqine, Leflunomide, Methotrexate, Minocycline, Sulfasalazine

Bone neoplasm treatments

Biopsies by skilled musculoskeletal tumor surgeons to excise margins and prevent spread; surgery; chemotherapy; radiation; multi disciplinary team approach including nurses, oncology, pain management; include patient and family in decision making

Bisphosphonates for bone metasasis

Bisphosphonates act in 2 ways: bind directly to mineralized bone matrix and limit bone resorption by osteoclasts; have a direct apoptotic effect on tumor cells. Several generations now available: 1st generation: clodronate; 2nd: pamidronate, ibandronate; 3rd: zoldronic acid. Multiple studies show efficacy in treating pain related to bone metastases and also reduction in incidence of skeletal related events eg, fracture.

A patient recently diagnosed with Paget's disease asks the NP, "What type of complications can occur as the disease progresses?"

Bone deformities of the skull place pressure on the cranial nerves, contributing to vertigo, hearing loss, and vision problems. Because vascular calcification can occur with Paget's disease, heart failure is possible. Hypercalcemia can lead to renal calculi. Kyphosis with advancing disease may affect cardiac and pulmonary function.

What test is most useful in determining the severity of bone metastasis?

Bone scan

CREST for dx of scleroderma

C - calcinosis of the skin, Ca++ deposits from under the skin leading to thickening of skin R - Raynaud's S - sclerodactyly - tightening of the skin T - telangieciases - dilated blood vessels Dx is made if the pt has 2 of the 5 symptoms. Up to 90% of pts have anticentromere antibodies present in the blood.

Limited scleroderma (CREST syndrome)

C- Calcinosis (CA deposits under skin) R - Raynaud's phenomenon E - esophageal dysfunction S - sclerodactyly (tightening of the skin) T - telangiectasia (dilated blood vessels) 2 out of 5 symptoms confirm the disese Anticentromere antibodies + < 90%

Suggested Ca++ & Vit D intake

Ca++: women under 50: 1,000 mg/d; women over 50: 1,200 mg/d; men under 71: 1,000 mg/d; men over 71: 1,200 mg/d; very low Ca++ intake associated with increased fx risk. Vit D: women and men: daily recommended intake: under age 50: 400-800 IU/d; age 50 & over: 800-1,000 IU/d; upper limit for daily intake all ages in 4,000 IU.

Pre-Op exam of RA patient

C1-C2 sublaxation Cricoaryternoid arthritis w adductor spasm of the vocal cords and a narrow airway. Pulmonary fibrosis. Risk for GI bleeding. Need for stress steroid coverage. Discontinue NSAIDs several days preop. Discontinue methotrexate 1-2 weeks preop** (coverage with analgesic meds or if necessary short-term, low dose steroid if RA flares)

Bone resorption (osteoclast products)

CTX (C-telopeptide of collagen cross-links) NTX (N-telopeptide of collagen cross-links)

Calcium pyrophosphate deposition disease (CPPD)

Can cause monarthritis clinically indistinguishable from gout - Pseudogout. Often precipitated by illness or surgery. Pseudogout is most common in the knee (50%) and wrist. Reported in any joint (including MTP -metatarsophalangeal). CPPD disease may be asymptomatic (deposition of CPP in cartilage).

Knee injury: rotational stress

Can cause: ACL sprain/tear, meniscus damage

Knee injury: hyperextension

Can cause: ACL tear, joint capsule sprain

Knee injury: varus stress

Can cause: LCL sprain, meniscus injury

Knee injury: valgus stress

Can cause: MCL sprain, meniscus injury

Knee injury: hyperflexion

Can cause: PCL injury; joint capsular sprain

Knee injury: fall on flexed knee

Can cause: PCL sprain

Knee injury: direct trauma

Can cause: contusion, fx, bursitis

Geriatric considerations

Careful PE; malnutrition suspected - CBC, albumin, cholesterol; UA, CBC, Chem panel, coag studies; lytes if on diuretic; EKG & CXR

Establishing a diagnosis of tumor/cancer

Careful medical history including constitutional symptoms - weight loss, night sweats, fevers, hair loss, pain at night; age; duration; other disease processes; previous toxic exposure or traumatic histories

Chondrosarcoma arises from:

Cartilage. These tumors are insidious and affects mostly adults over the age of 40 in the femur or pelvis

Avascular necrosis

Caused by distrupted blood supply leading to infarction of the bone. Results in degenerative arthritis. Most common in men and at the hip. Can occur in any bone. Risk factors: post-fracture; chronic steroid use; gout; alcohol abuse; sickle cell disease; chemo; radiation; deep-sea diving; use of compressed air.

The single largest cause of disability in children is:

Cerebral Palsy

Brachial plexus special tests

Cervical compression test; cervical distraction test; Spurling's test; brachial plexus traction test

)You are doing a preop physical for a routine THA on a 43 yo F w RA since age 20. PMH b/l TKA. No other medical problems. Current mes: NSAID, low-dose prednisone, MTX (methotrexate), and HCQ (Hydroxychloroquine, sold under the brand name Plaquenil). General PE normal. MS exam, extensive deformities, mild synovitis. In addition to routine tests, what test should be ordered before surgery?

Cervical spine films AP&Lateral, NOT flexion and extension. We don't want to miss a C1-C2 sublaxation. RA can cause asymptomatic instability of the neck - manipulation under anesthesia can cause spinal cord injury!

Burst fracture

Characterized by multiple pieces of bone; often occurs at bone ends or in vertebrae

Signs and symptoms of hypoparathyroidism

Chief complaints: mental fatigue; tiredness; abdominal pain; muscle cramps; Parkinson-like symptoms. PE: muscle spasms/tetany, excitability; deep tendon reflexes hyper; dry skin/hair loss; weakened tooth enamel (in children).

Who shouldn't take Teraparatide?

Children w open epiphyses; pt w h/o osteosarcoma or Paget's disease; pt w h/o High Beam radiation exposure therapy to the skeleton whether by implants or external involving potential skeletal exposure; pregnancy; hyperparathyroidism; any hyper-calcemic state; metastatic disease to bone.

Bone neoplasms seen in less than 20 year old

Chondroblastoma; Chondromyxoid Fibroma

Epiphyseal - tumors & cancers

Chondroblastoma; giant cell tumor; osteochondritis; Brodie's abscess; clear cell chondrosarcoma

Acute Sarcoid Arthritis

Chronic inflammatory disorder - noncaseating granulomas at involved sites. 15-20% arthritis; symmetrical: wrists, PIPs (proximal interphalangeal), ankles, knees. Common with hilar adenopathy. Erythema nodosum. Lodgren's syndrome: acute arthritis, erythema nodosum, bilateral hilar adenopathy

Salter-Harris Pediatric Fracture Classification

Classes are based on location of fracture relative to the growth plate. I: Transverse fx - epiphyseal separation II: Metaphyseal fx + epiphyseal separation III: Epiphyseal fx IV: Vertical fx through epiphysis + metaphyseal fx V: Compression fx of growth plate Amount of growth remaining will impact degree of remodeling.

Amputations

Classified as partial or complete. Most susceptible areas: distal foot, lower leg, fingers, forearm. Types: Guillotine-type (best success of re-implant); Avulsion/tearing (difficult re-implantation due to nerve/vessel damage)

General principles of pediatric fracture care

Clear hx - time and mechanism; determine site of injury; assess neurovascular status; obtain appropriate radiographs - 2 orthogonal views; immobilize; determine timeliness of specialty referral.

Osteomalacia: clinical complaints and PE

Clinical complaints: skeletal pain and tenderness; pain w ambulation; muscular weakness; generalized malaise. PE: spine may show kyphosis/scoliosis, vertebral collapse; deformities of weight bearing bones; waddling gait.

Thoracic outlet syndrome

Compression syndrome of upper limb neurovascular bundle at the level of scalene muscles and first rib. Special tests: Adson's test; Allen's test; Military brace position.

Paget's disease - clinical complaints, PE

Clinical complaints: variety of symptoms; longstanding pain symptoms; deep aching sensation w weight bearing; pain unrelated to activity; report of loss of height. PE: general: asymptomatic; conductive hearing loss; complications of CHF, Paget's Sarcoma. Orthopedic PE: kyphosis; bony deformity of skull; impaired fx healing; Pagetic sites are warm and tender.

Meniscus injury

Clinical features: pain at time of injury persists & interferes w wt bearing; tenderness of joint line; c/o locking and swelling; positive McMurray's/Apley's/Thessaly test. MRI or arthroscopy. Immobilize if painful. Crutches, quad exercises, NSAIDs. If remains locked, continues to give way and swelling persists, surgical referral.

When is bracing for scoliosis required?

Cobb angle > 20 20-40 Thoracolumbosacral orthosis (TLSO) bracing to prevent further curvature. 40 degrees - surgical fixation

Paralytic ileus

Common after bowel surgery but can occur after any surgery. Ileus most likely to occur after anterior lumbar surgery. Risks: Long anesthesia time, pain meds. Results in: Loss of peristalsis causing pseudo obstruction. S/S: N/V, abd distention, pain, lack of bowel sounds. Tx: NG tube, NPO until resolved. Monitor lytes, serial abd films. If severe, may need general surgery consult.

Geriatric guidelines

Complete H&P; cog assessment; depression screening; risk factors for postop delirium; substance abuse/dependence screening, ETOH; cardiac eval following American College of Cardiology/AHA algorithm for pts underdoing noncardiac surgery; risk factors for postop pulmonary complications and implementing suitable prevention strategies; functional status and fall history documented; calculate frailty at baseline; assessing nutritional status and considering implementation of preoperative interventions for high-risk patients; complete Rx hx, preop adjustments, monitoring for polypharm; identify patient's treatment goals and expectations; assess family and social support system; performing suitable diagnostic tests prn.

The NP suspects a patient has early disseminated Lyme disease. Which of the following should the NP expect to find in this patient? Conjunctivitis Arthritis Mild cognitive difficulty Distal paresthesia

Conjunctivitis. Ocular manifestations such as conjunctivitis have been reported in the early disseminated phase of Lyme disease. All other options represent findings associated with late disease.

Treatment of hyperparathryoidism

Conservative tx: adequate hydration (dehydration increases serum Ca++); avoid thiazide diuretics (increase serum Ca++); adequate Ca++ intake (1,000 mg/d; avoid high intake); control/prevent hyperphosphatemia in CKD (HyperPhos -> PTH); stabilize bone mineral density (BMD); encourage physical activity (stimulates bone mass); bisphosphonates (inhibit bone resorption & increase bone mass); DEXA scan every 2 yrs; calcimimetic meds (Sensipar/cinacalcet 30 mg BID); nonsurgical pts with severe hypercalcemia and/or normal BMD Symptomatic: Parathyroidectomy is an effective therapy that cures the disease, decreases the risk of kidney stones, improves bone mineral density, and may decrease fracture risk and modestly improve some quality of life measurements. **Can place part of gland in forearm: it will vascularize and function properly Watchful waiting: observing the patient over time with regular monitoring is reserved only for very mild cases, wherein blood Ca++ levels are only mildly elevated, there is no evidence of kidney damage and bone mineral density is normal or just minimally decreased. It requires diligent monitoring on the part of the patient and clinician with a minimum of twice yearly testing. Any change will require additional treatment.

Posterior shoulder dislocation

Findings: seizure and inability to passively or actively rotate affected arm externally

What test is used to help diagnose lateral epicondylitis of the elbow?

Cozen's: the pt is seated, the practitioner stabilizes the elbow while palpating the lateral epicondyle. The pt pronates and extends the wrist against resistance. Pain w motion &/or over the condyle is positive for tennis elbow. This test can be made even more selective by having the pt extend each finger against resistance while the wrist is flexed.

Femoral anteversion

Curvature of the femur. After 3 years old, resolves by 8 years old. Brace not effective. Osteotomy if functional problems after 8 yrs old.

Glenohumeral joint instability

Findings: shoulder pain in throwing athletes; anterior gelnohumeral joint pain and impingement

Polyarthritis

Definite inflammation (swelling, tenderness, warmth of >5 joints) A patient with 2-4 joints is said to have paucior oligoarticular arthritis

Immediate tx of malignant hyperthermia

DC INH agents, succinylcholine; hyperventilate w 100% O2; bicarb 1-2 mg/kg prn; get additional help; dantrolene 2.5 mg/kg IV push, repeat prn; cool patient: gastric lavage, surface, wound; tx arrhythmias; do NOT use Ca+ channel blockers

A 65-year-old female patient is referred for her first DEXA scan. The NP should explain that the patient's T score of -2.0 is concerning for:

DEXA scans are used to diagnose osteopenia and osteoporosis. A T score of -1.0 or more is considered normal. A T score of -1.0 to -2.5 is considered to represent osteopenia. A T score of-2.5 or lower is diagnostic of osteoporosis.

Postop steroid tapering

Day 1: hydrocortisone 100 mg IV q8h starting with induction of anesthesia Day 2: If patient stable and major postop stress resolved, lower dose of hydrocortisone to 50 mg q8h Day 3: Hydrocortisone 25 mg q8h Day 4: Hydrocortisone 25 mg BID Day 5: Maintenance dose (12-15 mg hydrocortisone/m2/d): 15-20 mg AM and 5-10 mg PM

Hypoparathyroidism

Decreased secretion of PTH leads to hypocalcemia. Etiology: acute/chronic; uncommon hormone deficiency; abnormal mineral absorption; 2nd to injury (surgery, trauma, or neoplasm); malabsorption. Incidence: children: idiopathic or autoimmune disorder. Adult: irreversible following surgery for hyperparathyroidism; acquired reversible 2nd to ETOH or malabsorption syndrome

RANK ligand inhibitor

Denosumab (Prolia) Mechanism of action: monoclonal antibody that links to RANK to decrease resorption. More BMD increase in hips and distal radius compared to alendronate. AE & SAE similar placebo effect. Quickly reversible. Safety: FDA warning increased BMD turnover and increased skin infection 0.2% over placebo. Indications: tx of postmenopausal women with osteoporosis at high risk for fx. Administered by health professional, 60 mg q6mo SQ. Contraindications include hypoCa++. Pre-existing hypoCa++ must be corrected prior to initiating therapy. Warnings: hypoCa++; serious infections including skin; dermatologic rxns: dermatitis, eczema; osteonecrosis of jaw; suppression of bone turnover - could include atypical femur fx after long-term use. Common adverse rxns: back, extremity & musculoskeletal pain, cystitis, hypercholesterolemia.

Which osteoporosis medication can be given to a patient with renal damage?

Denosumab is an effective osteoclast inhibitor, has a short half-life, and can be given to patients with compromised renal function.

Wolff's law

Developed by the German anatomist and surgeon Julies Wolff in the 19th century, states that bone in a healthy person or animal will adapt to the load under which it is placed. If loading on a particular bone increases, the bone will remodel itself over time to become stronger to resist that sort of loading. Basically, if you utilize bones, weight-bearing and mobility, the bones will be stimulated to remodel and grow. Wolff's law applies to many aspects of orthopedic health, injury recovery and joint replacement.

Paget's disease - diagnostics & labs

Diagnostic testing: plain radiographs; demineralization (early); bone overgrowth (late); typical mosaic pattern of bone growth; bone scan; metabolic activity. Labs: serum alk phos elevated; may also be elevated in metastatic bone disease; urinary hydroxyproline; reflect osteoclast activity

Hypoparathyroidism: differential, dx

Differential dx: pseudohypoparathyrodism; Vit D deficiency; malabsorption; kidney disease. Rx use: steroids, diuretics, antiepileptics. Dx labs: serum Ca++ low; serum phosphate high; PTH low; Vit D low; urinary Ca++ low. Radiographic findings: increased bone density.

Anterior shoulder dislocation

Dislocation of the humerus that occurs in the anterior inferior direction. Presentation: a visible deformity or out of place shoulder, swelling or discoloration (bruising), intense pain, inability to move the joint. May also cause numbness, weakness or tingling near the injury, such as in the neck or down the arm. The muscles in the shoulder may spams from the disruption often increasing the pain. Plan: attempt to reduce; sling the arm (above the heart); assess vascular integrity of the arm; assess neurological integrity of the arm; transport for immediate reduction. Acute dislocation: something breaks or tears and therefor can be repaired; repair better and easier than reconstruction. Chronic dislocation: instability has additional plastic deformation of the capsule and glenohumeral ligaments therefore needs to be shortened; restoring the normal functional anatomy is impossible.

Fell on arm or outstretched hand

Dislocation: deformity, check wrist pulse, sensation. Hyperextension: passive extension, valgus test, varus test.

The NP splints the arm of a patient with a distal radius fracture. The NP should suspect the fracture is a result of a vitamin D deficiency because of which patient comorbidity?

Distal radius fractures can indicate a bone disorder such as osteopenia, osteoporosis, or osteomalacia. Patients with celiac disease, diabetes, eating disorders, and other conditions that require a special diet may develop vitamin D deficiency.

Muscular dystrophy

Duchene's is the most common type of MD. Most patients are wheelchair-bound by 10 years. Only 25% survive beyond age 21. CPK will be 200-300 times normal. In addition to biopsy and EMG, EEG to evaluate cardiac problems, and pulmonary function tests are usually done. These children have loss of independent ambulation, soft tissue contractures and spinal deformity. They will have a wide stance, rocking gait d/t weak hip girdle muscles, and use a maneuver called Gower's sign to get up. Care will focus on symptoms involvement and complications. Parental support is needed on a community basis and to assist with depression and guilt, along with genetic counseling for the future.

Pathophysiology of concussion

During the minutes to days following concussion, damaged cells are not destroyed, but they are vulnerable to second impact, changes in cerebral blood flow, increased intracranial pressure and apnea. This is the result of a metabolic dysfunction resulting from the trauma. 'Metabolic cascade' following concussion: increased glucose demand; decreased cerebral blood flow; results in altered metabolic state & increased K+ & Ca++; the brain is more vulnerable to further injury during this cascade; this cascade occurs for up to 1 week after injury.

Osteomalacia: Dx, labs

Dx testing: radiographs: may see pesudo fracture (looser zone), compression fx, tunneling or pseudo-fx, bowing of long bones. Labs: decreased serum Ca++; decreased serum Phos; decreased Vit D; decreased 24hr urine Ca++ & Cr; Increased alk phos. Uncalcified bone found on biopsy.

Rotator cuff tear diagnostics and tx

Dx: radiographs may show decreased subacromial space, high riding humeral head, osteophytes; MRI shows tears. Tx: rehab, esp in older pts; surgery in younger, treat pain. Rehab: minimal in first 4 weeks (passive ROM); weeks 6-12 increase ROM active assisted, graduated weight training; return to sport: full activity 12 weeks.

Vertebrae

Each individual vertebrae consists of a vertebral body or centrum that transfers weight to the next lower vertebrae, a vertrebral arch forming the posterior margin of the vertebral canal, and variable types of processes that either provide attachment points for muscles or articulate with ribs. The 5 sacral vertebrae begin fusing after puberty and are usually completely fused by age 25-30. The coccyx is not completely fused until late in adulthood.

Clinical presentation of diffuse scleroderma

Edema of hands. Thickened skin w loss of skin folds. Stiff joints. Decreased thoracic excursion. H/O of Raynaud's. GI issue/esophageal dysfunction. Causes changes to internal organs. Anti topoisimerase-I or Anti-Scl-70 antibodies + in < 40%. CXR, PFTs may show changes.

Lyme Disease

Epidemiology: tick borne disease (Borrelia burgdorferi in deer tick); Multi-system, inflammatory; in 2010, 94% of cases from Northeast with cases noted in MN & WI. Early presentation: distinct skin rash - local lesion; recent h/o pt walking in wooded areas. Later stages: cardiac/neurologic/arthritis complications

anylosing spondylitis

Etiology: unknown - link to HLA-B27 gene; chronic inflammatory disease; axial skeleton; Synovial inflammation -> fibroblast response -> dense fibrous scars -> vertebrae fuse. Epidemiology: onset age 25-34 yo; occurs in males 5X greater than females. Clinical presentation: long h/o dull achiness w stiff back or neck, ascending stiffness of spine, fatigue and impaired sleep. Diagnostic testing: radiographs show bamboo spine (later stages, calcification formation between vertebra); elevated ESR, decreased PFT (pulmonary function test) if there is thoracic involvement. Pharamcologic tx: NSAIDs/DMARDS/AntiTNF blockers; non-narcotic analgesics prn; muscle relaxants prn. Non-Pharmacologic tx: balance activity and rest periods; PT; daily stretching exercises; assistive devices - canes

Diaphyseal - tumors and cancers

Ewing's Sarcoma; eosinophilic granuloma; osteoid osteoma

Limitations of drawing bone markers

Expensive and no current standardization for labs but research for this is ongoing. Various therapeutic interventions may affect release of some bone markers. Substantial work remains in characterizing existing assays, identifying better markers and performing the clinical studies to define which bone markers should be measured and when.

Middle finger jam resulted in severe pain and loss of ability to extend the distal phalanx of the finger indicates avulsion (torn away from the bone) which structure of the distal phalanx?

Extensor digitorum profundus tendon

Differential diagnosis in inflammatory arthritis

Eye involvement: conjunctivitis in reactive arthritis, uveitis in enteropathic and sarcoidosis, episcleritis in RA. Oral ulcerations: painful in reactive arthritis and enteropathic, not painful in SLE. Nail lesions: pitting (psoriasis), onycholysis (reactive arthritis). Alopecia (SLE)

Secondary trauma assessment

F - full set of vitals, monitoring; family presence G - give comfort measures - touch/verbal/reassurance H - history; Head to toe assessment I - inspection of posterior surface: worst damage can be underneath!

Shoulder pain

Fall down - pain on top of shoulder: A-C separation - Piano key sign; A-C shear test. Fall down - pain in front of shoulder: dislocation - apprehension test; axilla region palpation. Neck twisted - burning pain down arm: stinger - rule out other causes of pain; test strength around shoulder. Arm goes limp: sublaxation - apprehension test; relocation test. Pain on raising shoulder: rotator cuff - Empty can; full can; drop arm test.

Fat embolism

Fat released from bone or free fatty acids from adipose tissue after trauma. S/S occur from a couple of hours to 48 hrs after trauma including petechiae & hypoxia. Can affect lungs, heart, brain, kidnesy or skin. May see fat cells in urine, sputum or blood. Tx is symptomatic. May show changes in mental status. Petechiae of the skin and mucosa above nipple line and blanch. PO2 drops to less than 50 mmHg. CXR w diffuse 'snowstorm' effect. V/Q scan to r/o PE. Early recognition to prevent morbidity and mortality. Minimize movement of long bone fractures. Respiratory support (intubation & ICU if needed). High risk factors: long bone fracture, multiple fractures, pelvis fractures, rib fractures, total joint replacement, spinal fusion).

Symptoms of HPA suppresion

Fatigue; anorexia; n/v; abdominal pain; fever; hypoglycemia; hypotension; circulatory collapse; hyponatremia

Common traumatic fractures

Femoral neck - little old lady hip fracture. Tibial plateau - complex fracture at the knee. Tibial Plafond (pilon) - complex fracture at the ankle. Midshaft femur. Rib fx - flail chest. Wrist fx - falling forward and catching self with arms out front. Clavicle - sling, then mobilize shoulder after 1 week. Radial head - sling, unless comminuted, then operative.

Common injuries of unrestrained driver in MVA

Femur fx: 64%; Pelvic fx: 46%; chest injury 46%; ankle fx: 39%; facial bone fx: 37%; cranial injury: 16%.

Preop cardiac considerations

For emergency noncardiac surgery, preop heart testing should not be done; rather, the pt should immediately undergo the emergency procedure. Pts w severe or symptomatic cardiovascular disease and/or active cardiac conditions should undergo eval & tx before noncardiac surgery. These conditions include unstable CAD, blockage of 2 or more coronary vessels, decompensated HF, significant cardiac arrhythmias, or severe valvular disease.

Angulation fracture

Force is applied laterally causing angulated fracture

Skin traction

Force is applied to the skin; buck extension traction, Russell traction, Bryant traction

Tapping fracture

Forceful or direct blow causing fracture and soft tissue injury

What is a complication from crutch walking to monitor for?

Forearm muscle weakness is a sign of radial nerve injury caused by crutch pressure on the axillae

Injury: first hits hand on object

Fractured metacarpal: tender at site of fx; malrotation of fingers in fist.

Osteochondritis Dissecans

Fragment of cartilage and underlying bone is detached from articular surface. Mechanism of injury: repetitive trauma; usually radial head compression of capitellum. Pathology: AVN of capitellum. Incidence: 17-45% elbow pain in children and adolescent pitchers. Hx: lateral elbow pain, occasional catching and locking. Risks: repetitive micro-trauma from throwing; rapid growth; poor throwing technique. PE: tenderness over radiocapitellar joint; elbow effusion; limited ROM. Xray: radiolucency of entire capitellum, subchondral fragment.

Gout prophylaxis

Frequent attacks >3/year, tophi development or urate overproduction. Avoid use of meds that contribute to hyperuricemia: Thiazide and loop diuretics, low-dose salicylates, niacin, cyclopsorine, ethambutol (Losartan promotes urate diuresis and may even normalize urate levels. This action does not extend to other members of the ARB class. Useful in elderly with HTN + gout). Colchine 0.6 mg daily-BID. Use alone or in combination with urate lowering drugs. Prophylaxis w/o urate lowering drugs may allow tophi to develop.

38 yo male got his right ring finger caught in a player's shirt while playing touch football. Felt pop in his finger and now has pain. What are the differential dx?

Fx; DIP or PIP dislocation; extensor tendon injury; flexor tendon injury; collateral ligament tear

68 yo F w 3 yr ho RA new pt. Presents w 4 wks increasing fatigue, dizziness, dyspnea, and anorexia. Joint pain and stiffness are mild and unchanged. Managed with ibuprofen and hydroxychloroquine until 4 mo ago, when a flare cause a switch to piroxicam and prednisone. PMH: peptic ulcer 10 yrs ago, mild HTN. Exam shows a thin, pale, apathetic woman w T 98.4F, BP 110/65, HR 110. Symmetrical 1+ synovitis of the wrist, MCP, PIP and MTP joints. Exam of heart, lungs, and abdomen is unremarkable. What system much you inquire more about today?

GI** NSAID gastropathy is sneaky and can be fatal! Clues of impending disaster: high risk for NSAID gastropahty; presentation suggestive of blood loss (pale, dizzy, weak, tachycardia w low BP); no evidence of flare in RA to explain recent symptoms of increased fatigue

Side effects of bisphosphonates

GI: abdominal pain, n/v, diarrhea or constipation, may exacerbate existing GI problems (e.g ulcers). Lyte disturbances: hypocalcemia, hypophosphatemia. Fever and flu-like symptoms w IV admin, typically at the start of treatment. Impaired renal function. Atrial fibrillation w Zoledronic acid (Reclast). Atypical femur fracture.

Injury: thumb pulled back

Gamekeeper's thumb: UCL valgus stress test; pain on UCL

A patient presents with a bump on the dorsal surface of the right hand. The NP notes the prominence is located at the base of the second metacarpal joint. What should the NP suspect as the diagnosis? Paronychia Lipoma Giant cell tumor Ganglion cyst

Ganglion cyst. The dorsal and volar surfaces of the wrist often are associated with benign ganglion cysts. Paronychia are localized infections of the nail that can cause nail deformities; recurrent paronychia infection can be caused by underlying squamous cell carcinoma. Lipomas are soft and superficial, reasonably well-defined, and often occurring on the thenar eminence. Giant cell tumors are non-tender masses found near an interphalangeal joint (usually thumb, index, or long finger).

Metaphyseal - tumors and cancers

Giant cell tumor; simple cyst; fibrous dysplasia; osteoblastoma; chondromyxofibroma; osteosarcoma; aneurysmal bone cyst; non-ossifying fibroma

Continued tx of malignant hyperthermia after initial phase

Give dantrolene 1 mg/kg every 4-6 hrs for 24-48 hrs; monitor for recrudescence - rate is 25%; follow lyte panel, blood gases, CK, core temp, urine output & color; coag studies. Biochemical markers: blood gases esp p CO2, pH; myoglobin levels in serum and urine; PT, PTT, INR, fibrin split products, liver enzymes, BUN; monitor for signs of myoglobin and rhabdomyolysis and institute therapy to prevent renal failure.

52 yo M w destructive RA treated w NSAID and low-dose prednisone. MTX (methotrexate) started 4 mo ago, now 15mg/wk. Presents w 3-wk h/o fever, dry cough, and increasing SOB. Exam: low grade fever, fine rales in both lungs, normal CBC and liver enzymes, low albumin, diffuse interstitial infiltrates on CXR. What next?

Give oral steroid for hypersensitivity pneumonitis and stop methotrexate. DMARDs have a dark side. Methotrexate may cause serious problems: lung, liver, bone marrow. Be on the look out for toxicity w all the DMARDs

Paget's disease management

Goals: control symptoms; prevent long term complications. If asymptomatic: monitor patient at regular intervals. Conservative tx: if symptomatic: meds, assistive devices. Surgical intervention: correction of malalignment, bowing of long bones; total joint arthroplasty. Coordinated care approach: Ortho: ambulation w assistive devices, fx tx, TJA. ENT: conductive hearing loss. Rheumatologist: if not response to treatment. Neurologist: spinal stenosis, loss of function, hydrocephalus.

H2 blockers such as ranitidine (Zantac) will _________ availability of calcium carbonate

H2 blockers such as ranitidine (Zantac) will impair availability of calcium carbonate, which requires an acid environment to dissolve. Calcium citrate decreases the risk of kidney stones and dissolves at all pH levels.

HLA-B27

HLA-B27 is a blood test to look for a protein that is found on the surface of white blood cells. The protein is called human leukocyte antigen B27 (HLA-B27). Human leukocyte antigens (HLAs) are proteins that help the body's immune system tell the difference between its own cells and foreign, harmful substances.

Recommended preop testing

Hgb for major surgery w significant expected blood loss or CBC count if the cost is not substantially increased; Serum Cr level for people older than 40; EKG in pts older than 40; CXR in pts older than 60; no labs must be repeated if results were normal

malunion fracture

Healing in an abnormal position or alignment, occurs when there is improper alignment of fracture fragments.

Fibromyalgia ACR criteria

Heightened myodenia (pain to touch). History of chronic widespread pain >3 mo. Exhibit >11 of 18 tender points. ACR criteria are both sensitive (88.4%) and specific (81.1%)

Estrogen/Progesterone

Helps retain Ca++ and phosphate. Increases osteoblasts (rebuilders). Increases bone matrix. Levels decreased with aging.

Palliative radiation therapy

Highly effective therapy: relieves pain, stops tumor growth, can prevent fx, can be used after surgery for broken bones. Used for pain control in pts w uncomplicated painful vertebral metastases. Also indicated for pts w poor performance status and short life expectancy.

This means a patient is at 'high risk' for fracture in the next 10-yrs according the the FRAX Score

Hip FRAX risk score greater than 3% OR any site FRAX score greater than 20%

Athletic injury assessment: HOPS

History of injury Observation & inspection Palpation Special tests (test structural integrity; functional activity tests)

What are the primary causes of osteoporosis?

Hormonal imbalance, faulty metabolism and poor dietary intake of Ca++ cause primary osteoporosis. Secondary osteoporosis may be caused by alcoholism, malnutrition osteogensis imperfecta, RA, liver disease, scurvy, lactose intolerance, hyperthyroidism, and trauma.

Pathophysiology of hypoparathyroidism

Hormonal insufficiency of PTH. Multi-system involvement. Bones: decreased mineralization, decreased bone resorption. Serum hypocalcemia. Malabsorption: intestinal Ca++ absorption down. Mild metabolic alkalosis.

Phase I of bone remodeling

Hormonal/Biochemical; Physiologic Indicators; Activate precursors; Osteoclast Formation.

Internal validity

How much control you have over the study: history, maturation, testing, morality, instrumentation, selection bias.

Knee injury evaluation

Hx: unusual sound/sensations (clicking/locking: meniscus injury; 'pop': cruciate ligament injury, patellar dislocation); h/o previous injury/surgery. Inspection/observation: always compare bilaterally; obvious deformity: genu valgum ('knock knees'), genu varum ('bow legged'), genu recurvatum ('hyperextension'); bleeding; dislocation/ecchymosis; swelling (immediate vs gradual, amount); scars. Palpation: patella, femoral condyles, tibial plateaus, tibial tuberosity, fibular head, joint line (menisci), MCL, LCL, infrapatellar tendon, quadriceps, hamstrings, gastrocnemius.

Associated conditions for CPPD

Hyperparathyroidism Hypercalcemia Hypocalciuria Hemochromatosis Hypothyroidism Gout Aging

The NP is treating a patient diagnosed with hyperparathyroidism. The nurse should teach the patient this disorder interrupts the metabolism of calcium and phosphorus, contributing to development of which of:

Hyperparathyroidism causes bones to become fragile, contributing to development of osteoporosis.

5 stages of fracture repair

I: immediate - hematoma, clot, inflammation, bony necrosis II: 3-14 days - phagocytes begin the process of removing necrotic tissue III: 2-6 weeks - granular tissue and callus begins to form IV: 3-6 months - callus becomes ossified and is slowly replaced by trabecular bone V: 6 mo-1yr - remodeling

IV bisphosphonates use for bony metastasis

IV bisphosphonates are prescribed to reduce potential for pathological fracture, except in spinal cord compression. They are also prescribed to help manage the pain of bone metastasis. IV bisphosphonates decrease osteoclastic activity.

Early warning signs of malignant hyperthermia

Increase in end-tidal carbon dioxide (even w increasing ventilation), tachycardia, muscle rigidity, tachypnea, and hyperkalemia. Later signs include fever, myoglobinuria, and multiple organ failure (MOF)

Reasons for open approach to fracture management

If multiple fragments are present, the fx is close to the joint line, or soft tissue is lodged between the bony edges, the fx must be visualized by the surgeon. Linear fxs, non-displaced fxs and mobility below the level of the injury may indicate that an ORIF is NOT indicated.

Non-Pharm Tx for acute gout

Immobilization of joint. Ice packs. Abstinence of ETOH (consumption can increase serum urate levels by increasing uric acid production. When used in excess it can be converted to lactic acid which inhibits uric acid excretion in the kidney). Dietary modification (low carb, increase protein and unsaturated fats, decrease in dietary purine-meat and seafood. Dairy and vegetables do not seem to affect uric acid - bing cherries and vitamin C)

Developmental dysplasia of the hip (DDH)

In a child with DDH, the hip socket is shallow. As a result, the head of the femur may slip in and out. It may dislocate.

The NP student evaluates a patient with fatigue, muscle weakness, and bone pain. The patient recently had a normal DEXA scan. The student asked the NP what laboratory tests would be necessary to rule out osteomalacia. In addition to serum 25-hydroxyvitamin D, calcium, phosphorus, and parathyroid hormone, the NP should suggest the student order:

In addition to the identified tests, the NP should order alkaline phosphatase to rule out osteomalacia.

Treatment of Paget's disease:

In general, asymptomatic patients do not require active treatment, but rather careful monitoring. A symptomatic patient should be treated with NSAIDs and Calcitonin to reduce pain. Opiate meds should be avoided whenever possible. Bisphosphonates such as Fosamax and Actonel may be used to help retard bone loss. Quite often a pt requires the use of assistive devices such as a cane, walker and shoe lifts to help with ambulation. PT can also be useful in gait training.

Testosterone

In males converts to estrogen like effects on the bone. Stimulates osteoblastic activity. Stimulates osteoblastic activity. Helps with bone strength activity. Bone loss with aging increases w low T level.

Hormone therapy for bone metasasis

In women w breast CA, used for estrogen sensitive tumors. In men w prostate CA, androgen-depriving therapy results in greater bone loss over time.

Why is hypothermia during surgery a problem?

Increases infections, reduces oxygenation, increases skin pressure damage, can lead to increased pain and muscle pain d/t shivering and increases cardiac risk.

Adjunct to traction: balanced suspension w Thomas splint & Pearson attachment

Indications: Device that supports the extremity & overcomes the force of gravity; used with skin or skeletal traction for femur or tibial fractures not amenable to internal fixation, for acetabular fractures, for maintaining joint space following removal of a prosthesis and for hip/knee contractures; can be used alone for exercise, to support dependent part &/or to maintain correct alignment. RN: pad ischial ring; check increased pressure in groin and knee areas. Neurovascular assessment. Position the extremity in sling to keep pressure off the heel and Achilles tendon to not carry weight of lower extremity. Pearson attachment parallels knee. Prevent foot drop - use footboards. If bed in semi-Fowlers, lay flat at least 20 minutes every shift to prevent hip flexion contracture. Assure no external rotation of extremity to prevent peroneal nerve palsy. Perform quadriceps muscle setting exercise and heel cord exercises 10 times per hour while awake and prn.

Clinical use of bone turnover markers (BTM)

Indicator of bone quality. Prediction of bone loss in untreated postmenopausal women. Prediction of fractures in untreated postmenopausal women. Monitoring of response to therapy in postmenopausal osteoporosis. Use of bone turnover markers as a tool for optimizing adherence to therapy.

Complications of traumatic fracture

Infection (PPX w 48 hrs abx); compartment syndrome; chronic regional pain syndrome (CRPS or RSD); pressure ulcers; paralytic ileus; hemorrhage; DVT; PE; fat emoli; arthrodesis (fusion of joint d/t severe infection or if non-reconstructable); pseudoarthrosis (fracture site develops a fibrous connection instead of a bony connection); nerve injury (bony fragments, edema, compartment syndrome, many or may not be permanent); avascular necrosis (AVN); rhabdomyolysis

Reactive arthritis

Infection-induced systemic disease with inflammatory synovitis from which viable organisms cannot be cultured. Associated with HLA B27 Asymmetric, oligoarticular, knees, ankles, feet. 40% have axial disease (spondyloarthropathy). Enthesitis: inflammation of tendon-bone junction (Achilles tendon, dactylitis). Extraarticular: rash, nails, eye involvement

Acute Polyarthritis

Infection: gonococcal; meingococcal; lyme disease; rheumatic fever; bacterial endocarditis; viral (rubella, parvovirus, Hep B) Inflammatory: RA; JRA; SLE; reactive arthritis; psoriatic arthritis; polyarticular gout; sarcoid arthritis

Acute Monoarthritis

Inflammation (swelling, tenderness, warmth) in one joint Occasionally polyarticular diseases can present with monoarticular onset: RA, JRA, Reactive and enteropathic arthritis, Sarcoid arthritis, viral arthritis, psoriatic arthritis

Lateral epicondylitis

Inflammation of the muscle attachment to the lateral epicondyle of the elbow. Often caused by strongly gripping. Commonly called tennis elbow. Evaluation: pain w active & resistive extension of the wrist and forearm; pain and weakness w manual muscle test for the extensor muscle group; palpable pain over lateral epicondyle; positive tennis elbow test; positive Cozen's test. Tx: evaluate biomechanics w coach assistance; discuss racket tension; discuss grip size; U/S for inflammation; iontophoresis using dexamethasone if no improvement; ice cup after activity; strap to reduce tension on extensor tendons; steroid injections; may immobilize wrist in volar splint; role of PRP? Injection seems to be more effective than NSAIDs.

Dermatomyositis/Polymyositis

Inflammatory myopathy of skeletal muscles. S/S: fatigue; fall unrelated to balance; heliotrope rash; shawl sign; V-sign. Dx test: elevated CK (muscle damage); EMG - inflammation; biopsy - necrosis. Tx: Corticosteroids; immunosuppressives; IV IgG; topicals for rash; bed rest - slow return; ROM activities.

Chi-square

Involves categorical variables. Looks at 2 distributions of categorical data to see if they differ from each other.

Juvenile Idiopathic Arthritis (JIA)

JIA typically is marked by an insidious onset of arthritis symptoms in a child younger than age 16. One of the key points in diagnosis of JIA is the presence of arthritis. Arthritis must be consistently present for at least 6 weeks in order to establish the diagnosis of JIA.

Injury: finger bent back

Jersey finger: inability to flex the DIP joint. Volar plate injury: pain on volar surface of finger.

Post-concussive syndrome

Long lasting symptoms of concussions. More prevalent w multiple concussions. Symptoms vary greatly: mood changes, headache, memory/concentration problems, etc.

Types of mechanisms of injury

Kinetic/mechanical; thermal; chemical; electrical; radiant; oxygen depreivation

SLE diagnostic tests

LE cell; ANA titer; Anti-DNA; Complement fixation; ESR/CRP. Criteria to DX: Malar, discoid rash; photosensitivity; arthritis; renal disorder; immunological disorder; DNA, ANA

Gradual onset of pain after heavy use

Lateral epicondylitis: tennis elbow test, Cozen's test

A patient with rheumatoid arthritis is scheduled to undergo total knee arthroplasty with general anesthesia. The NP completes a history and physical at the pre-operative visit. What radiographic views of the cervical spine should the NP order?

Lateral flexion and extension. Lateral flexion and extension views of the neck may demonstrate C1-C2 instability secondary to erosion of the ligaments that hold the odontoid in place. These findings are important if the patient is having surgery that involves intubation or manipulation of the neck because it could lead to quadriparesis or death.

Stages of tissue breakdown: necrosis

Length of pressure: develops after 6 hrs of unrelieved pressure within microvascular collapse and thrombosis. Resolution: may require a vascular surgeon and/or wound care team consult.

Stages of tissue breakdown: ischemia

Length of pressure: occurs after 2-6 hrs of unrelieved pressure. Resolution: may require up to 36 hrs to resolve.

Stages of tissue breakdown: hyperemia

Length of pressure: occurs when pressure is applied less than 30 minutes. Resolution: within 1 hour

Stages of tissue breakdown: ulceration

Length of pressure: occurs within 2 weeks of necrosis. Resolution: may require a vascular surgeon.

Medial elbow pain in young pitcher

Little league elbow: medial epicondylitis test

Clues to look for C1-C2 sublaxation

Long-standing RA or JRA. May have NO symptoms. C2-C3 radicular pain in the neck and occiput. Spinal cord compression: quadriparesis or paraparesis, sphincter dysfunction, sensory deficits, TIAs secondary to compromise of the vertebral arteries

Assessment of a fracture

Look: ecchymosis, deformity, hemorrhage, swelling, abnormal skin indentations. Listen: crepitus, air from a punctured chest wall, breath sounds. Feel: abnormal movement, crepitus, pain, pulses and skin temp.

A patient presents to the clinic for pre-operative evaluation after completed laboratory testing. The NP identifies a low serum calcium; physical examination reveals neuromuscular irritability and muscle twitching with a positive Chvostek sign. The NP should suspect the patient has:

Low serum calcium can contribute to neuromuscular irritability, skeletal muscle twitches, and tetany.

Erythematosis, Lupus

Lupus is an autoimmune disease in which the body creates antigens that attack different body tissues. 3 types of lupus: Discoid, Systemic, and Drug-induced

In early June a 15 yo boy comes to your practice w his mother. He had been fine until about 5 days ago when he developed a fever. He has a stiff neck and a rash on his back. He and his family live in Connecticut near the New York State border. His mother reports he was playing in the woods recently. He has a bulls-eye rash on his arm. What is the diagnosis?

Lyme disease. Erythema migranes rash, along with fever and stiff neck, along with the geographic local of the patient suggests Lyme disease. Other s/s: fatigue, headache, muscle and joint pain. Borrelia burgdorferi is the causative agent of Lyme disease (spirocete, same family as syphillis). Treat with amoxicillin or doxycycline for 3-4 weeks.

Sidelines assessment tools for concussion

Maddock's questions Standardized Assessment of Concussion (SAC) Balance Error Scoring System (BESS) Support Concussion Assessment Tool 2 (SCAT2)

Nursing considerations with bisphosphonate administration

Maintain adequate calcium and vitamin D intake while on bisphosphonate therapy. Do not take oral bisphosphonates w food. Impaired absorption occurs when administered with antacids and mineral supplements that contain Ca+, aluminum, iron or Mag+. Avoid using w NSAIDs to reduce renal & GI adverse effects. Pre-treatment dental exams and good dental hygiene are recommended for patients on IV bisphosphonates to monitor for osteonecrosis of the jaw. Aminoglycosides may precipitate additive hypocalcemia effects when administered with bisphosphonates.

Traction principles

Maintain prescribed line of pull. Maintain continuous pull. Prevent friction. Identify and maintain countertraction.

Injury: finger bent forward

Mallet finger: inability to extend DIP. Buttonhole finger: inability to extent PIP.

Apprehension test

Maneuver: anterior pressure on the humerus with external rotation. Dx if +: anterior glenohumeral instability.

Neer's sign test

Maneuver: arm in full flexion. Dx if +: subacromial impingement

Drop-arm test

Maneuver: arm lowered slowly to waist from abduction Dx if +: rotator cuff tear

Speed's maneuver

Maneuver: elbow flexed 20-30 d and forearm supinated. Dx if +: biceps tendon instability or tendonitis

Yergason test

Maneuver: elbow flexed to 90 d w forearm pronated. Dx if +: biceps tendon instability or tendonitis

Cross-arm test

Maneuver: forward elevation to 90 d & active adduction. Dx if +: acromioclavicular joint arthritis

Hawkin's test

Maneuver: forward flexion of the shoulder to 90 d & internal rotation. Dx if +: supraspinatous tendon impingement

Relocation test

Maneuver: posterior force on humerus while externally rotating the arm. Dx if +: anterior glenohumeral instability.

Sulcus sign

Maneuver: pulling downward on elbow or wrist. Dx if +: inferior glenohumeral instability

'Clunk' test

Maneuver: rotation of loaded shoulder from extension to forward flexion. Dx if +: labral disorder

Spurling's test

Maneuver: spin extended w head rotated to affected shoulder while axially loaded. Dx if +: cervical nerve root disorder.

Closed reduction

Manipulation of an extremity to realign into correct anatomical position. Xray confirms alignment. Cast, splint, brace, buddy taping. External fixator or traction is then applied to maintain alignment. If open wound/contamination present will need I&D and abx coverage.

Concussion

May be caused by either a direct blow to the head, face, or neck or elsewhere on the body with an 'impulsive' force transmitted to the head. Results in rapid onset of short-lived impairment of neurologic function that resolves spontaneously. May result in neuropathologic changes, but the acute clinical symptoms largely reflect a functional disturbance rather than a structural injury. Results in a graded set of clinical symptoms that may or may not involve loss of consciousness. No abnormality on standard structural studies is seen in concussion.

criterion validity (predictive validity)

Measures predict "real world" outcomes -Sat scores predict success in college. Test instrument against an established instrument.

Little leaguer's elbow

Mechanics that lead to problem: stretching of medial joint structures: ulnar collateral ligament; compression of lateral joint structures: radius-capitellar joint. Hx: gradual worsening, elbow aching. PE: medial epicondylar tenderness; diffuse swelling; loss of full extension d/t pain. Xray often helpful: widening apophyseal line, capitellar OCD, olecranon physeal irregularities. May have avulsion fx from medial humeral epicondyle. Tx: Rest 3-6 wks followed by PT; NSAIDs; brace limiting ROM; loose bodies and locking, catching &/or elbow effusion require referral for arthroscopy.

A nurse approaches the NP to question laboratory testing for a patient with Paget's disease. The NP's response should be based on the knowledge the patient will need ongoing monitoring of:

Medical management is indicated to prevent further bone loss along. This should include laboratory monitoring of calcium, phosphorous, and vitamin D. Life-long monitoring of serum alkaline phosphatase (ALP) is required. CBC may be indicated during a perioperative period due to increased bleeding but is not routinely performed.

Bone neoplasm seen in older than 40 year old

Metastatic disease; chondrosarcoma

Temporal patterns in polyarthritis

Migratory pattern: Rheumatic fever, gonococcal (disseminated gonococcemia), early phase of Lyme disease Additive pattern: RA, SLE, psoriasis Intermittent: gout, reactive arthritis

Treatment for known or suspected adrenal insufficiency

Minor surgery: 100 mg IV hydrocortisone with induction followed by usual maintenance dose of 20mg/d Major surgery: 100 mg IV hydrocortisone with induction followed by same dose q8h x 24h

ImPACT (Immediate Post-Concussion Assessment and Cognitive Testing)

Modules: symptoms inventory (word learning, X's & O's, # tracking, visual span, symbol matching, word/color discrimination, 3 letters - trigrams). Domain: Self-reported symptoms (verbal memory, visual working memory, attention, visual attention span, processing speed/memory, reaction time, working memory/processing speed

Inflammatory Arthritis features

Morning stiffness >1 hr Fatigue: profound Activity: improves symptoms Rest: worsens symptoms Systemic: Yes Corticosteroid: yes

Noninflammatory arthritis features

Morning stiffness: < 30 min Fatigue: minimal Activity: worsens symptoms Rest: improves symptoms Systemic: no Corticosteroid: no

Spine fractures of bone metasasis

Most patients treated successfully with non-operative management and irradiation. Principal indications for surgical intervention: progressive neurologic compromise (decompression indicated with cord or root compression caused by retro pulsed bone or disc fragments; spinal instability causing neural compromise); intractable mechanical spine pain unresponsive to irradiation or bracing may benefit from kyphoplasty

Vertebrae fracture pearls

Mostly at cervical or lumbar area. High cervical fx - C1,2,3 are life threatening. Stable fractures are treated with continuous immobilization w cervical collar, TLSO, LSO, or corset braces. Unstable fxs are treated surgically via fusion, decompression, instrumentation. Non-operative management with halo or cervical tongs. Most common cause is MVA. Can potentially cause damage to surrounding organs, veins, and arteries. Hemodynamic stability is priority. Diagnosed by PE, CT, MRI or bone scan.

Arthritis of SLE

Musculoskeletal manifestation 90% Most have arthralgia May have acute inflammatory synovitis RA-like. Do not develop erosion. Other clinical features help with DD: malar rash, photosensitivity, rashes, alopecia, oral ulceration

ACR (American College of Rhuematology) criteria for RA diagnosis

Must have 4 out of 7: Early morning stiffness > 1 hr; > 3 joint arthritis; Symmetrical arthritis; Wrist, MCP, PCP arthritis; RA nodules; Rheumatoid factor +; X-ray changes: periarticular osteopenia/marginal erosions

muscular dystrophy (MD)

Myopathy. Progressive, hereditary, degenerative weakness of skeletal muscles. 2-3/10,000 males. Dx: muscle biopsy, increased CPK, abnormal EMG. Gower's sing: climb up legs w hands. Many types: Duchenne's most common and severe w <20 year life span. Becker's seen at 7 yo: < 40 year life span.

Lupus Criteria

Need 4 of 11: M - malar rash D - discoid rash S - serositis O - oral ulcers A - arthritis P - photosensivity B - blood abnormalities R - renal A - ANA antibiodies I - immune abnormalitis N - neurologic

Why smoking is bad for the bones

Nicotine slows the production of osteoblasts and impairs the absorption of Ca++.

treatment of systemic sclerosis

No cure. Avoid cold. Cease smoking. Skin care. Good dental care (Sjorn's syndrome common). PT - maintain ROM, facial mobility. GI management. Meds: modify disease/symptomatic treatment. ACEI for renal crisis. Vasodilators to increase blood flow to fingers, prevent/tx pulm HTN. Immunosupressants. Sometimes requires dialysis, kidney transplant.

Systemic sclerosis (scleroderma) care

No cure. Smoking and cold can exacerbate Raynaud's. Skin care of the face and hands are particularly important. Dental care is imperative, esp when Sjogren's Syndrome has been confirmed (immune system disorder characterized by dry eyes and dry mouth). Face and body exercise will help to maintain mobility. GI management will assist with esophageal dysfunction and hypomotility.

Hyperparathyroidism management

Nonsurgical: watchful waiting if asymptomatic and bone density normal; Ca++ levels twice per year and have other monitoring one per year; avoid lithium, thiazides; stay active; stay hydrated; maintain adequate Ca++ intake; Vit D 400-600 units daily. Rx therapies: avoid meds that adversely effect kidney function and Ca++ levels; calcimimetic meds; bisphosphonates; DEXA testing; HRT if benefits outweigh risks. Surgical tx: most common tx for hyperparathyroidism: traditional surgery. Minimally invasive, 95% effective.

Effects of caffeine

Numerous physiologic effects, including increasing aerobic endurance and strength as well as delaying fatigue among adults performing physical activity. At rest, caffeine can increase HR, BP, attentiveness, gastric secretion, & temp.

Complications of Paget's disease:

OA; fxs; neoplasms sarcoma are rare, 0.3%, osteosarcoma most common form; neuromuscular syndromes; joint disease - hip and knee most common; cardiovascular abnormalities; angioid streaks of the retina (linear disruptions of the Bruch membranes, with proliferative connective tissue emerging through the defects); Hashimoto thyroiditis; Dupuyten contracture (gradual tightening of tissue under the skin in the hand); chondrocalcinosis (pseudogout, Ca++ crystals deposit in the joints causing pain); osteogenesis imperfecta (brittle bone disease); osteoporosis

Conditions that confound DXA results

OA; soft tissue calcifications; overlying metal objects; previous fx; severe scoliosis; extreme obesity or ascites; vertebral deformities; osteomalacia. If a pt has any of these conditions, their DXA may not be all it appears to be. The machine may pick up that the bone is 'thicker' in areas of calcifications, bone spurs, foreign objects, adipose tissue, etc.

nonunion fracture

Occurs when fracture healing has not taken place 4-6 mo after the fracture occurs and spontaneous healing is unlikely.

Compartment Syndrome

Occurs when there is a significant increase in pressure within the compartment. Causes vascular, nerve and soft tissue injury leading to tissue hypoxia. Any compartment can be affected, but usually in the lower leg. Can be limb-threatening and requires immediate surgical intervention. Advocate for the patient - time is of the essence! Monitor pressure of compartment with transducer. Release pressure-bivalve cast, unwrap dressing, fasciotomy.

Collateral ligament injury

Often caused by direct trauma. Tearing sensation noted by pt. Valgus and varus tests w knee in 30 d of flexion. Tx: Grade 1-2 conservative measures: ice, elevation first 24-72 hrs; crutches w limited WB; immobilizer/hinged brace 7-14 d; lateral ligament injuries requires 4-6 wks bracing. Grade 3 often conservative but get referral to surgeon.

Progression and complications of systemic sclerosis

Once in remission - relapse is uncommon. Lung impairment - leading cause of death. Loss of hand grips. GI: hypomotility -> malabsorption/constipation; esophageal fibrosis -> dysphagia.

Articular fracture

One that involves a joint surface.

Onycholysis

Onycholysis is when a person's nail or nails detach from the skin underneath. Although not a serious health condition by itself, onycholysis can be a symptom of a potentially serious illness.

Phase III of bone remodeling

Osteoblasts form bone; create new bone in formed cavities.

Phase II of bone remodeling

Osteoclast resorb bone; creates cavities in cancellous and cortical bone.

Appearance of Pagetic bone:

Osteoclasts: bizarre appearance, excessive #; bone is disorganized, trabeculae is irregular and rimmed with osteoblasts; mosaic appearance of bone is formed by irregular, randomly arrayed lamellar bone; increased bone volume; marrow is replaced by highly vascular stromal tissue, which consists of all the parts which do not conduct the specific functions of the organ - i.e. connective tissue, blood vessels, nerves, ducts, etc. Stromal tissue functions to contribute to the body's support and movement.

A 33-year-old male describes a dull aching pain in his thigh that has continued for a few months. He states the pain is worse at night. X-rays show small areas of lucency in the femoral shaft. Which of the following should the NP suspect? Osteoid osteoma Bone cyst Eosinophilic granuloma Chondromyxoid fibroma

Osteoid osteoma An osteoid osteoma occurs primarily in people ages 10-35. It is located in long bones and the posterior spine, and causes night pain that is generally responsive to NSAIDs or aspirin. Bone cysts typically occur in patients under age 20 and are characterized by a central medullary lytic lesion. Eosinophilic granulomas occur most often in young patients ages 5-10, rarely in anyone over age 30. They are lytic and often aggressive. Chondromyxoid fibromas also occur in adolescence and appear on x-ray as internal calcifications.

A 60-year-old female presents to the clinic with complaint of fatigue, myalgia, and leg pain. Pseudo fractures are noted on x-ray of the distal femur. The patient reports she became a strict vegetarian after her recent gastric bypass surgery. Based on the patient's history and symptoms, which condition should the NP suspect as the MOST likely diagnosis?

Osteomalacia is the result of severe deficiency of Vitamin D often found in strict vegetarians and persons who underwent gastric bypass surgery or gastrectomy. Osteomalacia is characterized by myalgia, fatigue, muscle weakness, and bone pain with pseudo fractures most commonly noted at the distal femur.

Osteoporosis vs hyperparathyroidism

Osteoporosis: NORMAL serum Ca++, phosphate, PTH, & alk phos. Hyperparathyroidism: normal or LOW Ca++, phosphate, & PTH; LOW Vit D; HIGH alk phos.

Osteoporosis vs hypoparathyroidism

Osteoporosis: NORMAL serum Ca++, phosphate, PTH, & alk phos. Hypoparathyrodism: Ca++ levels rise as much as 25%; stomach upset; excessive urination & impaired kidney function; LOW TSH

Osteoporosis vs osteomalacia

Osteoporosis: NORMAL serum Ca++, phosphate, PTH, & alk phos. Osteomalacia: normal or LOW Ca++, phosphate, & PTH. LOW Vit D. HIGH alk phos.

The NP is discussing the use of chemotherapy with medical-surgical nurses on an adjacent unit. Which of the following should the NP identify as the type of tumor for which chemotherapy is MOST effective? Osteosarcoma Osteoblastoma Osteochondroma Osteoid osteoma

Osteosarcoma Chemotherapy for bone sarcomas (e.g., osteosarcoma, Ewing's sarcoma, malignant fibrous histiocytoma) has been shown to be effective. Osteoblastoma, osteochondroma, and osteoid osteoma are all benign bone tumors.

Shoulder pain causes

Overuse injuries - tendinitis, bursitis; injuries - tendon, cartilage, ligament tears, fractures; instability - traumatic/congenital; arthritis

Bone formation (osteoblast products)

P1NP (aminoterminal propeptide of type I collagen) P1CP (Carboxyterminal propeptide of type I collagen) BSAP (Bone specific alkaline phosphatase) OC (Osteocalcin)

Components of the pre-participation PE

PPE includes ht, wt, VS, eyes, ENT, heart, lungs, GI, genitalia (males only), skin, & MS. Clearance, the term used for permission for athletic participation, is divided into 3 categories: Unrestricted clearance; Clearance after completion of further evaluation or rehab; No clearance for certain types of sports or for all sports.

Common sites for bony metastases Mnemonic

PT Barnum Loves Kids P - Prostate; T - thyroid; B - breast, bladder; L - lung; K - kidney

80% of all skeletal metastases derive from:

PTBLK prostate, breast, lung, kidney, and thyroid cancers

6 P's of compartment syndrome

Pain (out of proportion to injury; with passive stretch) Paresthesias (nerve damage) Pallor (blood vessel damage) Pressure (intracompartment > 30 mmHg) Paralysis Pulselessness (late sign)

Inflammatory joint diseases features

Pain (when?): Yes (AM) Swelling: soft tissue Erythema: sometimes Warmth: sometimes AM stiffness: prominent Systemic features: Sometimes Increased ESR, CRP: Frequent Synovial fluid WBC: >2000 Ex: Septic, RA, SLE, Gout

Noninflammatory joint disease features

Pain (when?): Yes (PM) Swelling: bony Erythema: absent Warmth: absent AM stiffness: minor (<30') Systemic features: absent Increased ESR, CRP: uncommon Synovial fluid WBC: <2000 Ex: OA, AVN

Questions to ask: History helps in Differential Dx

Pain come suddenly, minutes? - fracture Over several hours or 1-2 days? - infectious, crystals, inflammatory arthropathy. History of IV drug abuse or a recent infection? - septic joint. Previous similar attacks? - crystals or inflammatory arthritis. Prolonged courses of steroids? - infection or osteonecrosis of the bone.

Labral disorder

Pain or "clunking" sound with overhead motion

Clinical presentation of a fracture

Pain; edema; discoloration; deformity; loss of normal function; crepitus; muscle spasms; protruding bone

Treatment for bone metastasis

Palliative radiation therapy: standard of treatment for pain and prevention of morbidity and disease progression. Limited application of hormone therapy (does not help with pain). Surgical stabilization for prevention of pathologic fx addresses pain and mobility issues. Bisphosphonate therapy.

What is the earliest sign of compartment syndrome?

Paresthesia. Other symptoms that may appear early on include pallor, weakened pulse, and swelling.

Pathophysiology, diagnostics, & interventions of delayed fracture healing

Patho: infection; inadequate fracture immobilization; inadequate blood supply to fracture site. Dx: serial xrays, CT, MRI Interventions: bone grafting, internal fixation, external fixation, electrical bone stimulation.

Psoriatic arthritis clinical findings

Patients may have 'sausage digits', asymmetric swelling, synovitis, and erythema in small peripheral joints, along with pitted, discolored nails. Treatment is aimed at symptoms relief w meds to manage psoriasis. NSAIDs, DMARDs, & BRMs (biologic response modifiers) are used to manage the erosive arthritis in some patients. Laser tx can be used for some. PT/OT is consulted for rehab and plans for ADLs. Radiology exam is used to monitor joint damage.

Common injuries of unrestrained front seat passenger in MVA

Pelvic fx: 46%; femur fx: 41%; cranial injury: 24%; abd injury: 13%

ACTH (Adrenocorticotropic hormone) for acute gout

Peripheral anti-inflammatory effects and induction of adrenal glucocorticoid release. 40-80 IU IM followed by second dose if necessary

The NP is developing a grant proposal to study the lived experience of persons with systemic sclerosis. Which of the following should the NP consider as an appropriate design? Phenomenology Ethnography Narrative analysis Discourse analysis

Phenomenology Phenomenology is an approach to understanding everyday life experiences; this method would be appropriate to explore the lived experience of persons with systemic sclerosis. Ethnography involves the description and interpretation of cultural behavior. Narrative analysis focuses on story as the object of inquiry to explore how individuals make sense of events in their lives; narratives are viewed as the medium for their experiences. Discourse analysis (conversation analysis) seeks to understand the rules and structure of conversations and texts.

Complications of external fixation

Pin tract infections (most common); loss of alignment or correction; joint stiffness; contractures; delayed healing (nonunion, malunion)

Athletic injury diagnostics

Plain xrays r/o fx, dislocation CT - fine bony detail, internal organs MRI - soft tissues (muscles, tendon, ligament, etc) Nuclear bone scan - stress fx, abnormal bone metabolism Labs - UA, CBC, etc, as indicated

Teriparatide (Forteo)

Portion of parathyroid hormone First drug to stimulate new bone formation. Injection only. Indications: tx of postmenopausal women w osteoporosis at high risk of fx; increase in bone mass of men w primary or hypogonadal osteoporosis at high risk for fx; tx of men and women w glucocorticoid-induced osteoporosis at high risk for fx. High risk defined as: h/o osteoporotic fx; multiple risk factors for fx; pts who have failed or are intolerant to other available osteoporosis therapy. Mechanism of action of PTH: binds to cell surface G protein-coupled receptor; decreased apoptosis of osteoblasts; stimulates differentiation of bone lining cells and preosteoblasts to osteoblast; result is net increase in # & action of bone forming osteoblasts.

Athletic injury assessment: observation

Position of body; look for swelling, deformity; skin discoloration, muscle spasm, eye movement, facial expression; compare to opposite side of body.

The NP reviews the results of related diagnostic studies completed before meeting with a 70-year-old patient with complaint of bone pain. Multiple lytic lesions also are noted on the x-rays. Which of the following results should the NP recognize as MOST consistent with multiple myeloma? Plasma cell filtration 3% in bone marrow Positive Bence-Jones protein in urine Positive HLA-B27 Elevated D-dimer assay

Positive Bence-Jones protein in urine Bence-Jones protein is considered a tumor marker for multiple myeloma and thus this finding is most specific. Plasma cell filtration in bone marrow is normally 2%-3%; in multiple myeloma, abnormal plasma cells make up at least 10% of the bone marrow. Human leukocyte antigen (HLA)-Cw5 or HLA-Cw2 may play a role, but not HLA-B27 (positive in inflammatory conditions such as reactive arthritis, psoriatic arthritis, ankylosing spondylitis). D-dimer elevations are generally related to conditions of hypercoagulability, such as DVT, HF, MI, and sepsis.

A 21 yo M presented w acute pain and swelling of 1 knee. On exam, the joint is tender and restricted in movement. Xray of the knee showed periarticular OA. On investigation, he had a raised ESR of 102, and mild anemia (Hb 10.6) but no detectable serum RA factor. The knee effusion was aspirated; the fluid contained a polymorphonuclear leucocytosis but no organisms or RA factor. No diagnosis was made at this stage but he was treated empirically with indomethacin; his arthritis improved. What was his diagnosis?

Possibly Ankylosing spondylitis. Our 21 yo M presented w acute pain and swelling of 1 knee returns 15 mo later and now has developed an iritis in his left eye, low back pain and stiffness. His peripheral joints were normal but pain could be elicited in both sacroilian joints. X-rays of his pelvis showed the classic changes of ankyosing spondylitis and tissue typing revealed that he was HLA-B27 positive. He has intermittent backache over the last 5 years, although daily exercises have limited the stiffness. He has developed bony ankylosis between the lumbar vertebrae.

Osteoporosis follow-up

Postmenopausal women and men age 50 and older. Repeat BMD testing per National Osteoporosis guidelines. Should have repeat DXA every 1-2 years if on med & 1-5 yr interval based on initial risk. No EBR guidelines for DXA timing currently. Routinely repeating DXA is not helpful in managing osteoporosis according to some. Healthy older women repeating BMD up to 8 yrs later added little value in predicting future risk according to 1 study.

Brachial plexus strain (Burner/Stinger)

Presentation: sudden force causing head to go one way and shoulder the other way, resulting in immediate pain, numbness and tingling down the affected side. tx: light stretch of injured area; perform upper extremity neurological assessment; light cervical traction; brachial plexus tension tests. Assess grip strengths. Ice over brachial plexus. Return to play: once full sensation and full strength demonstrated.

Psoriatic Arthritis

Prevalence of arthritis in Psoriasis 5-7% Dactilytis ("sausage fingers"), nail changes. Subtypes: asymmetric, oligoarticular associated dactylitis; predominant DIP (distal interphalangeal) involvement - nail changes; polyarthritis "RA-like" - lacks RF (rheumatoid factor) or nodules; arthritis mutilans - destructive erosive hands/feet; axial involvement - spondylitis - 50% HLAB 27 (+); HIV-associated - more severe

Concussion guidelines/goals

Promote use of uniform terminology. May lead to more conservative management of high school and college athletes. Promotes an awareness of concussion by non-specialist. Disadvantages: not based on scientific criteria. May be too limiting, particularly w professional athletes. Possibly assign too much importance to LOC. Do not predict severity or extent of recovery.

P-RICE-MM for acute injuries

Protection (stabilize area) Rest Ice Compression Elevation Medication (APAP, NSAIDs, analgesics, muscle relxants) Modalities (Dx tests, PT)

Lyme Disease

Pt hx: exposure to tick bite - wooded areas. Dx test: ELISA test screen for antibodies (once positive, will always be positive even after treatment). Tx: Antibiotics - PCN/Rocephin; NSAIDs; intra-articular corticosteroids for Lyme arthritis; medical specialist monitoring for systemic conditions.

The NP plans to conduct a Delphi study of nurse experts to identify essential competencies in management of patients with psoriatic arthritis. If the NP wanted to use experts representing all 50 United States, what sampling method should the NP use?

Purposive. Purposive sampling uses researchers' knowledge about the population to make selections. The researcher would select NPs from each state with known expertise in psoriatic arthritis management. In random sampling, each element in the population has an equal, independent chance of being selected. Consecutive sampling involves recruiting all people from an accessible population who meet eligibility criteria over a specific time interval or for a specific sample size. Quota sampling involves the researcher's identification of population strata with determination of how many participants are needed from each stratum.

What type of study will answer the question about how patients felt about having pain and nausea after surgery?

Quantitative

Rheumatoid factor

RF, an IgM antibody is seen in the serum of 75% of pts with RA. High titers of RF are associated with severe disease. RF is also found in other diseases like syphilis, sarcoidosis, infective endocarditis, TB, leprosy, parasitic infections; in advanced age and in asymptomatic relative of pts with rheumatoid disease. Antinuclear antibody are seen in 20% of pts with RA, though their titer is lower than in SLE.

A 57-year-old man complains of severe pain on his right heel and the bottom of his right foot. Other complaints include itching, drainage, and pain in his right eye consistent with conjunctivitis. He also has sterile pyuria. The NP should recognize these symptoms as consistent with:

Reiter syndrome. Reiter syndrome involves a symptom triad of conjunctivitis, urethritis, and enthesitis. Inflammation of the Achilles tendon and plantar fascia cause the most painful symptoms.

Gout Complications

Renal failure (ARF can be caused by hyperuricemia, chronic urate nephropathy Nephrolitiasis Joint deformity Recurrent Gout

What is the leading cause of death in patient's with SLE (systemic lupus)?

Renal failure. Renal failure is the leading cause of death in patients with SLE. GI effects are common, ranging from mild anorexia to life-threatening bowel obstruction. CNS complications include headaches, dizziness, memory loss, and mood disturbances. Approximately 75% of persons with SLE develop kidney damage within 2 years of diagnosis. As a result of renal involvement, pharmacologic treatment of hypertension is indicated.

When to refer to ED for concusion

Repeated vomiting; severe or progressively worsening headache; seizure activity; unsteady gait; slurred speech; weakness or numbness in the extremities; unusual behavior; signs of basilar skull fx; AMS w Glasgow Coma Score of <15.

Biceps Tendonitis

Swelling, inflammation in biceps tendon; pain upper, anterior shoulder in biceps groove; pain w shoulder, elbow flexion; pain w lifting; weakness d/t pain

Rhabdomyolysis

Results from muscle injury in which myoglobin is released into circulation causing acute renal failure. Large soft tissue injury or crush injury is at increased risk. Changes in BMP: increased K+, phosphate, CK, BUN, Cr, altered Ca+. Tx: aggressive hydration w 0.9% NS, dialysis if severe.

Falls prevention and safety

Risk for fx in a fall: failure to break a fall; falling to the side; age; low bone mass; unfavorable bone geometry; high bone turnover. Risk factors for falls: muscle weakness; poor balance; poor eyesight; poor overall health.

Risks and safety measures for lateral positioning

Risks: body prominence and pressure points on dependent side, spinal alignment. Safety: axillary roll for dependent axilla; lower leg flexed at hip; upper leg straight, with pillow between legs; padding between knees, ankles, and feet; maintain spinal alignment during turning; padded support to prevent lateral neck flexion; make sure ear isn't trapped.

Risks and safety measure of prone or face down position

Risks: head, eyes, nose, chest compression, iliac crest, breasts, male genitalia, knees, feet. Safety: maintain cervical neck alignment; pillow or folded towel under shoulders; protection for forehead, eyes, chin; padded headrest to provide airway access; chest rolls (clavicle to iliac crest) to allows chest movement and decrease abdominal pressure; knee free from torsion; padded with pillow to feet; padded footboard.

Risks and safety of beach chair position

Risks: negative pressure gradient between the surgical site and heart (venous air embolism); hypotension (cerebral hypoxia); neuropraxias lesser occipital, greater auricular, supraorbital. Safety: avoid venous air; for each inch of height between BP cuff and brain = 2 mm Hg drop in BP; proper position in tongs/head rest; avoid pressure on eyes.

Risks & Safety measures of Supine or Flat Position

Risks: occiput, scapula, thoracic vertebrae, olecranon process, sacrum/coccyx, calcaneus, patella, possible neural injuries: brachial plexus, ulnar, pudendal. Safety: padding to heals, elbows, knees, spinal column; occiput alignment with hips, legs parallel, and uncrossed ankles; armboards at </= 90 degree angle, parallel with floor, nor overlapping table edge; armboard pads level with table pads; head in neutral position.

Osteomalacia management

Rx: correcting Ca++, phos; calcitrol; phosphate supplements; Vit D supplements (PO, IV, sunlight). Treatment considerations: referral to treat/correct underlying conditions: hyperthyroidism, intestinal disorders, CKD. Surgical intervention: orthopedic surgery. Correction of long bone deformities. Kyphoplasty for compression fx of spine.

Salter-Harris Classification

SALTER-HARRIS - Physeal Injuries (SMACK - Same, Metaphysis, Articulation, Continuous, Krush) (SALTR - Same, Above, Lower, Through, Really bad) 1 - fracture through physis 2 - fracture through physis into metaphysis 3 - intra-articular fracture through physis into epiphysis 4 - intra-articular fracture through epiphysis, physis, and metaphysis 5 - crush injury

Rotator cuff

SITS Supraspinatus Infraspinatus Teres Minor Subscapularis

A patient with rheumatoid arthritis has warm swollen, and boggy joints. In addition to rheumatoid nodules, the NP should recognize what as a commonly occurring extra-articular manifestation of RA?

Scleritis. A patient with RA can present with multiple ocular symptoms, including dry eyes, corneal ulcers, and scleritis. Although vascular inflammation can occur, atherosclerosis is not related to RA. Although renal disease can result from RA, renal calculi and type 2 diabetes do not result directly from RA.

Scoliosis

Scoliosis is a sideways curvature of the spine that occurs most often during the growth spurt just before puberty. While scoliosis can be caused by conditions such as cerebral palsy and muscular dystrophy, the cause of most scoliosis is unknown.

Sequelae of concussions

Second impact syndrome Post-concussive syndrome

Rotator cuff tendinitis

Swelling, inflammation in rotator cuff muscles; pain upper, outer shoulder; overhead, rotary motions painful; pain at night; weakness d/t pain

Labs to evaluate for osteoporosis

Serum Ca++ (usually normal in osteoporosis) 25-hydroxy-vitamin D (calcidiol, calcifidiol) TSH, free T3, free T4, PTH, T level in men, protein electrophoresis (if suspicion for MM), Alk phos. May also consider if indicated: 1-25 dihydroxy-vitamin D (calcitriol); FSH; PINP & serum NTX or urine NTX; phos; bone specific alk phos (BSA).

Diagnostic testing for hyperparathyroidism

Serum Ca++ levels (>10 mg/dL); urinary Ca++ (24 hr); PTH radioimmunoassay (80% of patients) - PTH is usually low when Ca++ is elevated so if elevated think hyperparathyroidism; DEXA bone density. Rule Out: kidney stones; hypophosphatemia; hypochloremia.

The patient is experiencing an acute flare of rheumatoid arthritis. In addition to an elevated erythrocyte sedimentation rate, what laboratory results should the NP expect?

Serum albumin is expected to DROP in the acute phase inflammation; normal values are 3.5-5.5 g/dL. Platelet count is expected to INCREASE during an acute RA flare; this normal value (150,000-400,000 per liter) does not indicate active inflammation. Normal CRP is below 3.0; this value does not reflect active inflammation. Although ANEMIA can occur in patients with RA as a result of inflammation, this hematocrit value is normal (45%-52% for men, 37%-48% for women).

As part of the hospital's quality efforts, the Nurse Practitioner Fracture Liaison (NPFL) identifies patients with osteoporosis who are at risk of refracture. Which should the NPFL identify as critical to achieving targeted performance outcomes?

Setting clinical strategic goals. The Nurse Practitioner Fracture Liaison (NPFL) role can be instrumental in osteoporosis care management. The NPFL functions as a care manager to facilitate patient-centered coordinated care to close care gaps and improve treatment adherence. Setting realistic clinical strategic goals is imperative. The overwhelming workload of continuous patient follow up can have a negative effect on program success. Although professional associate support is important, it is not critical to achieving performance outcomes. Healthy People 2020 contains recommendations related to osteoporosis screening and hip fracture, but adoption of these objectives alone will not contribute to program success.

Return to game/practice after ankle injury

Sidelined 2-6+ weeks. Return when athlete can: balance on injured ankle, raising up on toes; run in a straight line; running changing directions; all activities must be pain free; hop 20 times on affected ankle w/o pain.

Tourniquet safety

Sizing of width and circumference; soft padding, not wrinkled; don't exceed 60 minutes UE, 90 min LE, 75 min peds LE.

Halo traction, indications & nursing considerations

Skeletal tongs: Vinke, Gardner-Wells, Barton, Crutchfield. Indications: cervical and high thoracic fractures, sublaxations, dislocations, fusions, scoliosis, maintain stability during surgery. RN: provide continuous pull (20-30 lb wt). If pin loosening or penetration occurs, apply manual traction & support sides of head w sandbags or apply Philadelphia collar. Notify surgeon asap. Incorporate s/s of cranial nerve impairment w neurovascular assessment. Observe for eye movements, pupillary changes, blurry vision, photosensitivity, difficulty swallowing, speech, or tongue control. Administer pin care every shift. Log roll. Muscle setting exercises and active ROM in appropriate joints. Provide distraction, social/recreational activities, emotional support. Skin assessment beneath vest. Allen wrench on vest for emergency use.

Pelvic sling traction

Skin (suspension) traction. Indications: Pevlic fx. RN: sling compresses side of pelvis. Provide good perineal and lower back care. Check for foot drop, voiding difficulties, perineal irritations, skin breakdown, correct size, fit, and application of sling. Maintain sling beneath lower back with cheek of buttocks elevated 1-2 inch from the bed. Clarify orders for lifting and turning. 10-35 lb wt effective.

Dunlop traction

Skin or skeletal traction. Indication: supracondylar fx of the elbow. Humerus fx. RN: Same as side arm traction. Used with children. Check for Volkman's ischemic contracture. 5-7 lb weight for humerus. 3-5 lb wt for forearm.

Side arm traction/90-90 upper extremity traction

Skin or skeletal traction. Indications: supracondylar fx of the elbow, humerus & shoulder. RN: do not change position in bed (back lying position) or position of head of bed. Tilt pt toward affected extremity. Maintain 90-90 traction w shoulder & forearm flexed or shoulder abducted & elbow flexed. Countertraction can be applied by placing shock blocks under the traction side of the bed. Observe for radial, ulnar, and median nerve pressure, numbness, and tingling in one or more fingers; decreased ability to oppose thumb and fingers (signs of Volkmann's ischemic contracture). Patient can use his/her feet to lift buttock for bedpan, skin, and changing of bed. Back and skin care for the upper portion of the body is accomplished by pressing down on the mattress. 2-3 weeks callus formation may be sufficient to allow for spica cast application. Provide emotional support and teaching about the traction and future care management. Provide social and recreational activities.

SLE (systemic lupus erythematosus) symptoms

Skin rash Arthritis Fever Anemia Joint pain Photosensitivity Hair loss Ulcers in the mouth Kidney damage

Cotrel's traction

Skin traction (combination head halter and pelvic belt). Indications: preopreative tx to help staighten spinal curvatures before insertion of skeletal rods for corrections of scoliosis. RN: pull in opposite directions helps to overcome deforming muscle pull causing curvature. Similar care as cervical head halter and pelvic belt traction. 1-2 hrs on, 1-2 hrs off for sleep. 5-7 lb for head halter. 10-20 lb for pelvic belt.

Buck extension traction

Skin traction used primarily for short-term immobilization or for correction of contractures or bone deformities, such as Legg-Calve-Perthes disease. Indications: fx of the hip for short term, hop contractures, muscle spasms from surgery (hip/knee) or arthritic conditions of hip/knee. RN: Keep pt's heels off the bed. Use pillow and/or heel protectors. Avoid pressure on the dorsum of the foot and over the head of the fibula or malleolus. Asses for peroneal nerve palsy. 5-7 lbs wt effective. Countertraction may be applied by elevating the foot of the bed to prevent sliding down and shearing or by slightly elevating the knee. The leg in traction should be on the mattress without a pillow under the leg. Place rolled towel or padded sandbag along the external surface of knee to prevent external rotation of affected leg. If on an unrepaired fracture, do not release the traction. Teach pts to use trapeze and unaffected foot/leg to lift themselves in assisting with bedpan use, skin care, and linen changes. In most instances, head of bed may be elevated for meals, but not for continuous positioning.

Russel traction

Skin traction. Indications: fractured hip; short-term use; fractured femur not amenable to internal fixation; fracture of tibia/fibula. RN: modification of Buck w the addition of a sling under the femur, not affected knee, to provide more comfort and less rotation. Hips and knees slightly flexed 30 degrees or less and immobilized. 2-5 lb wt effective. Make sure the sling is smooth and doesn't apply pressure in the popliteal space or the head of the fibula. D/t arrangement of the pulleys, the pull of traction is double the amount of wt applied. The arrangement of the ropes, pulleys, and knee sling distributes the pull more effectively throughout the entire limb, therefore less injurious to skin. Back-lying position.

pelvic belt traction

Skin traction. Indications: muscle spasms associated w low back pain, ruptured disc. RN: apply the pelvic belt across the pt's lower abdomen, making sure it is not on the pelvis and that it does not go above the iliac crest or umbilicus, and directly on the pt's own skin, when possible. Make sure of the correct size. Amount of wt varies from 15-40 lbs, depending on pt's size and amount of muscle spasms. Assure even/straight pull along thighs/knees to the spreader bar or weight attachment. William position (supine w 15-20 degrees elevation of the knees and 30-45 degree elevation of the head) preferred. If necessary, bed flat when side lying. Observe skin for irritation, heat and redness. Massage iliac crests. Instruct pt to ask for assistance to remove wts and belt if allowed for bathroom privileges.

Cervical head halter

Skin traction. Indications: severe sprains, strains, torticollis, mild cervical trauma. RN: observe for pain & pressure in the ears, TMJ, chin, & occiput. Add a soft, thin, foam pad beneath chin. Perform baseline cranial nerve assessment & document baseline findings. Men should be clean shaven when possible. May be set up so head is kept in a straight position or so that head of bed can be elevated depending on the patient's condition. As long as the patient's spinal column remains in correct alignment, patient is unable to change position, with assistance. Use of small cervical pillow determined by patient's condition. Elevate HOB 20-30 for correct alignment. Traction removed for meals if permitted by surgeon. Intermittent cervical skin traction is sometimes used per surgeon orders.

Bryant traction

Skin tractions (Gallow's traction in UK) Indications: developmental dysplasia of the hip (DDH); femur fx in child younger than 2-3 yo, weighing <30-45 lbs. RN: Make sure pt's buttocks just clear the mattress, with hips flexed 90 degrees and knee extended. Assure that spreader bar keeps pressure from being applied to malleoli. Be sure traction is taken down daily (every shift & PRN) to provide skin care of both extremities. Child may be positioned either parallel or perpendicular to head and foot of bed. If positioned perpendicular, then caregiver may bring child out of bed to hold in arms and feed. Toddlers may need to be restrained to prevent 'flipover'. Family may be taught to provide care for home Bryant traction, a 2-4 weeks period w 2-4 lbs of wt, followed by hip spica cast if necessary.

What is the most influential factor in trauma cases?

Speed

Ortho Trauma Emergencies

Spinal fx: suspected Pelvic fx: open book Dislocations: elbow, knee Amputations: partial/complete

Spondyloarthropathy

Spondyloarthropathy or spondyloarthrosis refers to any joint disease of the vertebral column. As such, it is a class or category of diseases rather than a single, specific entity. It differs from spondylopathy, which is a disease of the vertebra itself.

What is the most common organic cause of back pain in the young athlete?

Spondylolysis - a defect or stress fracture in the pars interarticularis of the vertebral arch. The vast majority of cases occur in the lower lumbar vertebrae (L5), but spondylolysis may also occur in the cervical vertebrae

Mechanisms of spinal fractures

Sports, MVA, axial loading, rotation, hyperextension, hyperflexion, penetrating injuries.

Sprain vs strain

Sprain involves tearing of the ligaments Strain involves tearing of tendon/muscle

Balanced Suspension Skeletal Traction

Steinmann pin or Kirschner wire 20-35 lb wt effective. Perform pin site cleansing every 38 to 72 hours with chlorexidine solution daily. Client does NOT wear boots. weights should hang freely.

The most common athletic cervical spine injury:

Stingers/Burners (brachial plexus strain). Associated with contact sports. Secondary to nerve root/brachial plexus neuropraxia. Symptoms of pain and paresthesia in the involved extremity. Mechanism: stretch or bending of the neck away from the affected side; compression or bending toward the affected side.

Risk factors for osteoporosis related fracture

Strong risk factors for fracture: menopause <45 yo; glucocorticoids; immobilization; BMI <19; anorexia nervosa; propensity to fall; malabsorption; CKD; transplantation; hypogonadism. Moderate risk factors for fracture: RA; ankylosing spondylitis; anticonvulsants; Ca++ intake <500 mg/d; DM; estrogen deficiency; primary hypoparathyroidism; smoking; ETOH abuse.

Hypothalamic-pituitary-adrenal (HPA) axis suppression following steroid use

Suspect this in any patient with persistent postop hypotension refractory to IVF who has received more than 20 mg prednisone daily (or equivalent of glucocorticoid) for more than 3 weeks in the previous year; OR any patient with clinical Cushing's syndrome (from any dose of steroid).

Patterns of joint involvement

Symmetric polyarthritis involving small and large joints: viral, RA, SLE, one type of psoriatic (the RA-like) Asymmetric, oligo- and polyarthritis involving mainly large joints, preferably lower extremities, esp knee and ankle: reactive arthritis, one type of psoriatic, enteropathic arthritis. DIP (distal interphalageal joint, the first knuckle from the top of the finger) joints: Psoriatic

Rheumatoid Arthritis

Symmetric, inflammatory polyarthritis, involving large and small joints. Acute, severe onset 10-15%; subacute 20%. Hand characteristically involved. Acute hand deformity: fusiform (tapering at both ends, spindle shaped) swelling of fingers due to synovitis of PIPs. RF may be negative at onset and may remain negative in 15-20%. RA is a clinical diagnosis, no lab test is diagnostic, just supportive.

Synovitis

Synovitis is the medical term for inflammation of the synovial membrane. This membrane lines joints that possess cavities, known as synovial joints. The condition is usually painful, particularly when the joint is moved. The joint usually swells due to synovial fluid collection.

The T and Z of DXA

T-score: the number of standard deviations above or below the mean for young Caucasian adults. Z-score: the number of standard deviations above or below the mean for the population matched to the patient in age, sex, race, and weight (used for less than 50 yo if not menopausal).

Acute Monoarthitis Etiology

THE MOST CRITICAL DIAGNOSIS TO CONSIDER: INFECTION! Septic Crystal deposition (gout, pseudogout) Traumatic (fracture, internal derangement) Other (hemarthrosis, osteonecrosis, presentation of polyarticular disorders)

Reliability

Test-retest - does instrument remain constant over time? Internal consistency - all subparts measuring same characteristics - an r value closer to 1 show smore consistency between items Equivalence - often called interrater reliability

Graduated Return to Play per the 3rd International Conference on Concussion in Sport

The 1st step in the graduated return to play approach is no acitivty, followed by step 2, which is light aerobic activity. Progression through the 6-step exercise program should occur with 24hrs between stages. If any symptoms recur while progressing through the program, the athlete should return to the previous asymptomatic level and progress again after another 24 hr rest period. Athletes should contact their PCP if symptoms recur. Typically, a minimum of 5 days is required to consider a full return to competition.

The NP evaluates a 55-year-old postmenopausal woman with back pain. Based on the National Osteoporosis Foundation guidelines, what other findings in this patient should prompt the NP to order bone mineral density (BMD) testing?

The National Osteoporosis Foundation (NOF) recommends BMD testing for postmenopausal women younger than age 65 who have one or more of the following risk factors: family history of osteoporosis, personal history of low-trauma fracture at age older than 45, current cigarette smoker, or low body weight (less than 127 pounds).

The NP is treating a 55-year-old patient with osteoporosis. When the patient asks about the daily calcium requirement, the NP should identify what total mg/day?

The daily calcium requirement for persons age 50 or older, regardless of gender, is 1200 mg.

Content validity

The degree to which the content of a test is representative of the domain it's supposed to cover. Usually determined by experts in the area of interest.

Pituitary gland

The endocrine system's most influential gland. Under the influence of the hypothalamus, the pituitary regulates growth and controls other endocrine glands. Responsible for: ACTH/TSH/FSH/LH and Prolactin. Peptide hormones regulate bone growth.

Construct validity

The extent to which there is evidence that a test measures a particular hypothetical construct. Test the instrument in situations where you expect to have differences.

A patient presents with recent onset of thigh pain. The NP notes the patient is limping and has tenderness to palpation over the femur. Radiographs of the femur show an osteolytic lesion with adjacent irregular overgrowth of bone and a mosaic pattern. An additional bone scan shows intense uptake at the site of the lesion. Based on these radiological findings, the NP should diagnose which of:

The primary diagnosis of Paget's disease is confirmed with these radiological findings. Osteolytic lesions are mostly found on the skull and long bones (e.g., femur). Adjoining overgrowth of bone is irregular in shape and x-rays show a characteristic mosaic pattern. Bone scans can assist with diagnosis and show intense uptake. Gait abnormalities may occur; patients report bone pain and focal areas are tender to the touch.

Indications for arthrocentesis

The single most useful diagnostic study in initial evaluation of monoarthritis: SYNOVIAL FLUID ANALYSIS -Suspicion of infection -Suspicion of crystal-induced arthritis -Suspicion of hemarthrosis -Differentiating inflammatory from noninflammatory arthritis

Parvovirus B-19

The virus of "fifth disease", erythema infectiosum (EI). Children "slapped cheek"; adults flu-like illness, maculopapular rash on extremeties. Joint involved more in adults (20% of cases). Abrupt onset symmetric polyarthralgia/polyarhtritis with stiffness in young women exposed to kid with E.I. May persist for a few weeks to months.

DMARDs (disease modifying anti-rheumatic drugs)

These drugs need frequent monitoring: blood, liver, lung and kidney are frequent sites of adverse effects. Interval of lab testing varies with the drug - 4-8 wk intervals are common. Most pts need to be seen 3-6 times/yr

Slipped Capital Femoral Epiphysis (SCFE)

This condition usually occurs in 10-16 year olds during rapid growth, when even minor trauma can precipitate its development. The epiphysis appears shorter and the epiphyseal plate wider, with smaller margins.

Blount's disease

Tibia vara, often asymmetric (50%). Higher rates in obese Black. Rx: 6-month bracing; surgical intervention if >30 mo w/o tx or structural/functional deformity: corrective proximal osteotomy; hemiepiphysiodesis; external fixator - some w gradual correction.

Transcondylar fracture

Transverse fractures of the condyles of the humerus or femur in which the fracture line crosses the fossae. Fracture extends into the joint capsule

Mallet finger

Traumatic avulsion of extensor tendon at insertion, DIP joint. May be associated w intraarticular fx. Fx involving >25% of articular surface need more than a splint. Those w volar subluxation need ORIF.

A patient with fibromyalgia syndrome complains of difficulty falling asleep. Which of the following should the NP recommend as MOST effective for initiating sleep in patients with this syndrome? Amitriptyline (Elavil) Temazepam (Restoril) Trazadone (Desyrel) Lorazepam (Ativan)

Trazadone (Desyrel) For sleep initiation, trazadone 25 mg is recommended. Temazepam and lorazepam are benzodiazepines, which are not recommended. Tricyclic anti-depressants such as amitriptyline are most effective for sleep maintenance rather than sleep initiation; they also are not always tolerated.

Subperiosteal

Type of implant with a metal frame that is placed under the periosteum but on top of the bone

Common peds fractures

Upper extremity fx accounts for 65-75% of all peds fx: clavicle, elbow region (supracondylar humerus, lateral condyle), radius and ulna

Gout Dx

Uric Acid (limited value as majority of hyperuricemic pts will never develop gout; levels may be normal during acute attack) CBC (mild leukocytosis in acute attacks, but may be higher than 25,000/mm) ESR (mild elevation or may be 2-3 x normal) 24 hr urine uric acid (only useful in pts being considered for uricosuric therapy or if cause of marked hyperuricemia needs investigation) Trial of colchicine (Positive response may occur in other types of arthritis to include pseudogout

Ottawa ankle rules

Used when deciding if imaging for lateral ankle sprain is appropriate. Order radiography if there is pain in malleolar region and 1) bone tenderness at tip or distal 6cm of the posterior edge of either malleolus *OR* 2) Unable to bear weight 4 steps.

Athletic injury assessment: Functional activity tests

Use only for mild injuries. Use the injured part to perform skill. Begin w very simple skills (walking, reaching, etc) and progress to very complex skills (running & cutting, etc). Begin each skill at half speed and progress to full speed. Do not allow any activity that causes pain. Determine if athlete can perform normally: observe performance, looking for smooth, fluid motion; Q athlete concerning pain or other abnormal feeling.

Gout prophylaxis: Urate lowering drugs

Used for documented urate overproduction. Goal is for serum urate concentartion to 6mg/dL or less. Start of therapy can precipitate acute attack; therefore, may need to use colchicine as long as six months. -Xanthine oxidase inhibitors: Allopurinol. Blocks conversion of xanthine to uric acid. Works for underexrectors and overproducers. Start typically 300mg/day and titrate weekly 100mg/day until optimal urate levels achieved. Start lower doses with renally impaired pts. -Uricosuric drugs: Probeneicd or Sulfinpyrazone. Increased renal clearance of uric acid by inhibiting tubular absorption. SE may prohibit use (GI and renal). Need measurement of 24hr urine in anyone for whom Probenecid therapy is initiated.

Corticosteroids for acute gout

Used for pts who cannot tolerate NSAIDs, or failed NSAID/colchicine therapy. Daily doses of prednisone 40-60 mg a day for 3-5 days then taper 1-2 weeks. Improvement seen in 12-24 hr.

Bone xray pearls

Used to confirm fx, joint space changes, osteochondral defects, calcification, effusion, spurs, loose bodies. May or may not see fx on initial xray; if pain continues, immobilize and repeat in 2 weeks. Multiple views are usually necessary. MRI, CT, or bone scan may pick up on fx missed by xray.

Methotrexate initiation in RA

Usually given after other meds have been tried w/o successful tx (i.e. NSAIDs) of symptoms. A dose is considered low - usually starting at 7.5 to 10 mg/week. If needed, the dose can be increased incrementally up to a max of 25 mg/week. Common s/e: changes in menstrual periods; vomiting, upset stomach; headache; dizziness, tiredness; blurred vission

Signs and symptoms of hyperparathyroidism

Vague at first - pt may report weakness, lethargy, depression, anorexia, and constipation. Other findings might include mental and personality changes, cardiac dysrhythmias, weight loss, and urinary calculi.

A patient with gout returns for follow-up examination by the NP, who reviews the patient's recent nutritional history. The NP should suggest the patient include which of the following as part of the daily regimen to help prevent an acute gout attack? Purines One liter of fluids Vitamin C White fish

Vitamin C. Vitamin C is beneficial in the prevention and management of high uric acid. Decreased consumption of foods with high purine content is recommended for persons with gout. Patients should also drink more than 2 liters of fluid daily. Because all types of fish contain purines, intake should be limited.

Patient education for Lyme disease

Walk in the center of path in the woods in high risk areas; wear light colored clothing so ticks are easier to see; wear long pants tucked into socks; check for ticks; use repellant; PROMPT eval for suspicious symptoms

The NP evaluates the x-rays of a patient who wrenched his knee in a fall on the ice. The NP identifies a cartilage-capped bony projection on the metaphysis of the distal femur as osteochondroma. The NP should educate the patient concerning which expected initial treatment?

Watchful waiting. An osteochondroma rarely causes symptoms and is often found incidentally on x-rays. Watchful waiting is the most common treatment. Wide excision at the base of the lesion may be needed, but it is not the initial treatment. Because osteochondroma is a benign lesion, radiation and chemotherapy are not indicated.

History of athletic injury

Where does it hurt? point When did it happen? Sudden or gradual? How did it happen? If hit, how? Position of body part when and after it happened? Former injuries. Level of training.

External validity

Whether the results of a study can be generalized to other situations and to other people. Protected for by randomization of sample and control of situational variables

The NP identifies bilateral femur fractures on the x-rays of a 3-year-old patient. The x-rays also show bones that are bent, with thickened cortices on the concave side. The NP should identify these findings as suggestive of which of:

With rickets, long weight-bearing bones are bent and the cortices are thickened on the concave side.

Splinting fractures/dislocations

Wrist fractures: sugar tong-splint. Metacarpal fx: ulnar or radial gutter splints encasing 2-3 digits. Digital fx: gutter splint vs alumifoam splint.

Pittsburg Knee Rules

Xray any of the following: - blunt trauma or fall as MOI + either of the following: -- age >55 or <12 -- inability to walk 4 wt bearing steps

Slipped Capital Femoral Epiphysis (SCFE) presentation

Young obese adolescent children (males>females) who complain of gradual onset of groin, knee, or hip pain often associated with a limp. However, some will have a painless presentation. Trendelenburg gait with external rotation. Xray both hips - 20% occur bilaterally. The 'ice cream' is falling off the ice cream cone.

Viral arthritis

Younger patients Usually presents with prodrome, rash History of sick contact Polyarthritis similar to acute RA Prognosis good; self-limiting Ex: Parvovirus B-19, Rubella, Hep B & C, acute HIV, Epstein-Barr virus, mumps

An RN questions the NP about osteoporosis medications. The NP should identify which of the following as the only drug that can be administered intravenously? Zoledronic acid (Reclast) Alendronate (Fosamax) Raloxifene (Evista) Teriparatide (Forteo)

Zoledronic acid is given intravenously once a year. Teriparatide is given subcutaneously every day for up to 2 years. Alendronate and raloxifene are taken orally.

Enchondroma

a benign tumor orginating from the cartilage.

Osteocyte

a bone cell, formed when an osteoblast becomes embedded in the matrix it has secreted. OsteoCYtes = 'cycle' of bone

Complete fracture

a break that extends through the entire thickness of the bone, often displaced

Osteoblast

a cell that secretes the matrix for bone formation. 'bone builders'

SERM (selective estrogen receptor modulator)

a drug that acts like estrogen in some tissues but like an anti-estrogen in others. Evista; Raloxifene. Indicated for prevention. Enhances beneficial effects of estrogen w/o increasing risks to breast/uterus. Caution use in pts at risk for DVT.

Achondroplasia

a dwarfing syndrome caused by gene mutation that causes abnormal endochondral ossification. It is a chronic condition. Males have a higher incidence than females. Longitudinal growth is most affected. C-spine injuries can occur related to poor head control. Progressive spinal stenosis results in associated complications.

Intracapsular fracture

a fracture inside the joint capsule

Extracapsular fracture

a fracture outside or not involving the joint capsule

Linear fracture

a fracture running parallel to the length of the bone; fracture line is intact; fx results from minor to moderate force applied directly to the bone

Longitudinal fracture

a fracture that runs parallel to the long axis of the bone

Ehler Danlos Syndrome

a genetic disorder caused by a mutation in the connective and muscle tissue genes. There are ten known variants of the syndrome. Classically, pts will demonstrate hypermobile joint, hyper-elastic skin, and easy bruising. There are no definitive diagnostic tests, although DNA linkage analysis may be done to differentiate the variant type of EDS. There is no cure and no treatment. Nursing care focuses on support.

Chondrosarcoma

a malignant cartilage lesion

Standard deviation

a measure of variability (how spread out values fall), average amount of deviation of values from the mean

Non-ossifying fibroma

a non-malignant tumor found near the end of the long bones and occurs in adolescents and children

A toddler noted to be bow-legged and in-toeing defect when walking reflects:

a normal torsional defect and 98% correct themselves. Tibial torsion can be internal or external and is a normal part of in utero development. As a result, torsion can be in-toeing or out-toeing. 0-20 degrees is normal at birth and most correct themselves by age 8. Observational management is the only treatment necessary unless it persists. A key part of treatment is to invite the grandparent to the clinic visit because they often become upset when they notice this. They may need reassurance and a reminder that their own son/daughter had the same. A key nursing diagnosis is Anxiety and is managed with good family education.

Stress fracture

a small crack in the bone that often develops from chronic, excessive impact

Multiple regression

a statistical technique that computes the relationship between a predictor variable (independent variable) and a criterion variable (dependent variable), controlling for other predictor variables (confounding variables)

Tumor

abnormal mass of tissue Benign: giant cell tumors; osteoid osteoma; osteochondroma. Malignant: osteosarcoma (arises from bone); chondrosarcoma (arises from cartilage); liposarcoma (arises from fat and muscle)

Meds contributing to bone loss

aluminum antacids; anti-seizure (dilantin, phonbarbitol), aromatase inhibitors (armidex, aromasin, femara); chemo drugs; cyclosporine A; guclocorticoids (cortisone, prednisone); GnRH (lupron, zoladex); heparin; lithium; depo-provera; methotrexate; PPIs (nexium, prilosec, prevacid, etc); SSRI; Tamoxifen (premenopausal use); synthroid (if too much)

Osteogenesis Imperfecta

an inherited connective tissue disorder that affects bone and soft tissue. It has an autosomal dominant or recessive gene. Multiple types, some of which can be fatal. Epiphyseal and articular cartilage are normal as well as bone minerals, but bone is generally osteopenic d/t thin, frail, sparse bone trabeculae. There is increased vascular fragility. *BLue Sclera, Shepard crook deformity, deafness

The shoulder is most often dislocated in which direction?

anterior (85%). Usually d/t indirect force, such as from abduction and external rotation. Humeral head lies inferior and medial to the glenoid.

Periarticular

area surrounding a joint

Mean

average, sum of all values divided by the # of values

Exercise for those with osteoporosis

avoid exercises involving forward flexion of the spine or bending forward from the waist. Avoid exercises that twist or jerk the spine. Participate in low impact exercise programs that allow one foot to remain on the floor at all times. Work w PT to develop a safe and appropriate exercise program.

Compression vertebral fracture:

axial compression usually stable if vertebral alignment remains

Greenstick fracture

bending and incomplete break of a bone; most often seen in children

Oblique fracture

bone break in which fracture line runs along an angle to shaft of the bone, 45 degrees

Comminuted fracture

bone breaks into many fragments

Displaced fracture

bone ends are out of normal alignment

Nondisplaced fracture

bone ends retain their normal position

Compression fracture

bone is crushed

Genu Varus

bow legged. Distance between knees if >2.5 cm with ankles together, external torsion of the femurs. Improves spontaneously by 18-24 mo. Increased in early ambulation and heavy children.

Fracture lines

can be classified by their angle relative to a bone's axis: linear, transverse, and oblique; spiral, comminuted, segmental, avulsed, impacted, torus, greenstick

Gout presentation

caused by too much uric acid in the blood (hyperuricemia). In 90% of cases, an excess production of uric acid or decreased renal function can account for the development of gout. Tophi nodules form and when they erupt, the onset is sudden and painful. It generally affects the lower extremity - usually the great toe. There is warmth, pain, swelling and extreme tenderness (symptoms called podagra). The pain usually starts at night. Even light pressure from the bed sheets are too much to take. The patient can run a fever in excess of 103 degrees and have a headache. In later stages, TOPHI FORM in the earlobes, fingers, hands, and feet. There is a heredity component to the disease. It is primarily seen in males between 4-60 years. In women it occurs more often after menopause.

Mild hypothermia increases the risk of surgical site infection by:

causing decreased delivery of oxygen to the wound space, subsequent impairment of the functions of phagocytic leukocytes and vasoconstriction

Osteoclast

cell that breaks down bone for resorption into blood supply (i.e. harvesting Ca++ in response to low serum Ca++, removing damaged cells, etc). 'clean up' cells.

Scheuermann's Disease

changes in vertebral end plates and disc space that can occur during development and lead to kyphosis. Often diagnosed in adolescence.

Finger sprain complications

collateral sprains: deformity, loss of function. Volar sprains: Swan-neck deformity (DIP hyperflexion, PIP hyperextension); pseudoboutonniere deformity (DIP hyperextension, PIP hyperflexion)

Longitudinal data

collect data at more than one point in time

Cross sectional data

collect data at one point in time

Mixed osteolytic and osteoblastic bone destruction and formation

common in breast cancer that has spread

Transverse fracture

complete fracture that is straight across the bone at right angles to the long axis of the bone, 90 degrees

Foods high in Ca++

dairy, white beans, spinach, broccoli eggs, sardines, and kelp. Low in Ca++: rasberries, papaya, pork, chicken, asparagus, carrots, kiwi, squash

Osteoporosis

defined as a skeletal disorder characterized by compromised bone strength predisposing to an increased risk of fx. Bone strength: reflects the integration of 2 main features: bone density and bone quality. Bone quality: refers to architecture, turnover, damage accumulation (microfractures) and mineralization. Bone density: or BMD refers to the mineral matter of the bone per square centimeter, used as an indirect indicator of osteoporosis & fx risk.

Histiocytic cell type to tumor/CA

develop into HistiocytosisX, lymphoma, giant cell tumor

Chondroblasts to cancer

develop into chondroblastoma, chondrosarcoma, osteochondroma, enchondroma

Fibroblasts to cancer

develop into fibromastosis, fibroma, non-ossifying fibroma, fibrosarcoma, fibrous dysplasia

Mixed tissue to tumors/CA

develop into malignant fibrous histiocytoma, chondromyxoid fibroma, adamantinoma, simple cyst, aneurysmal bone cyst

osteoblasts to cancer

develop into osteosarcoma, osteoid osteoma, osteoblastoma, ossifying fibroma

Signs of DDH (Developmental Dysplasia of the Hip)

difficulty abducting the hip, one leg appearing shortened, unequal skin folds, and difficulty positioning the legs for diapering are all signs of DDH

Signs of cerebral palsy

difficulty swallowing, scissoring of the legs, abnormal muscle tone, and tremors with muscle movement are all symptoms associated with Cerebral Palsy.

Jammed finger

dislocations of the IP joints occur most frequently at the DIPs; hyperextension in combination with axial compression causes dislocation of the distal phalanx and may rupture the palmar plate.

Slipped Capital Femoral Epiphysis (SCFE) defined

displacement occurs related to disruption of growth plate; the displacement may be stable or unstable.

Thyroid gland

endocrine gland that surrounds the trachea in the neck. Responsible for: calcitonin, T3/T4, serum Ca++ levels. Stimulates growth, fat and bone metabolism.

hyperparathyroidism

excessive levels of parathyroid hormone. Primary: PTH, hypercalcemia. Etiology: Often asymptomatic; parathyroid adenoma; thyroid hyperplasia; endocrine disorders. Secondary: results from another disease causing low Ca++; eventual hypercalcemia. Etiology: often asymptomatic; chronic RF; malabsorption syndrome; Vit D deficiency; osteomalacia; rickets.

kyphosis

excessive outward curvature of the spine, causing hunching of the back.

Acromioclavicular separation

falling on point of shoulder - pushes acromion down, acromioclavicular ligament tears - sometimes with major trauma coracoclavicular ligament tears. Grade I-II: sling, ice, pain meds. Grade III: may need surgery.

Parathyroid glands

four small glands on the posterior of the thyroid gland. Release PTH, influenced by LOW serum Ca++ levels, affected by Vit D 25-hydroxy level, activates kidney to resorb Ca++

Pathologic fracture

fracture caused by diseased or weakened bone

Impacted fracture

fracture in which one bone fragment is pushed into another

Supracondylar fracture

fracture of the distal end of the humerus; it is common in children and occurs when the child falls on the outstretched hand with the elbow partially flexed and may injure the median nerve

Avulsed fracture

fracture that occurs when the fracture pulls bone and other tissues away from the point of attachment

Segmental fracture

free floating piece of bone between well-defined fracture lines

NSAID gastropathy

gastric ulcers are more common than duodenal ulcers. No reliable warming signs. 80% of serious events occur w/o prior symptoms. Risk of hospitalization for NSAID ulcers in RA in 2.5 to 5.5 X higher than general pop. 107,000 pts are hospitalized and 16,000 death occur annually in US because of NSAID-induced GI complications

Digital dislocations that cannot be easily reduced:

generally have volar plate dislocation and need ORIF

Osteogenesis Imperfecta (OI)

genetic brittle bone disease. Dx: clinical deformities; blue sclera, Shepherds crook deformity; dentinogenesis, deafness. Diagnostics: radiographs, biopsy, bone density. Tx: brief light weight immobilizer; IM rods "Bailey" grows w child; Pamidronate treatment; pain meds; "brittle baby" precautions: NO BPs, don't pull on limbs; physical NOT mental handicap; encourage independence.

Falls evaluation

h/o circumstances surrounding the fall(s); drugs, acute or chronic medical problems, mobility baseline; examination of vision, gait and balance, function of the leg joints; examination of basic neurological function, including mental status, muscle strength, peripheral nerves of the legs, proprioception, reflexes, and tests of cortical, extrapyramidal, cerebellar function.

cortical bone

hard, dense, strong bone that forms the outer layer of bone; also called compact bone

Stellate fracture

have breakage lines radiating from a central point.

Burst vertebral fracture

heavy axial loading resulting in fragments that may press on nerves or spinal cord

Ewing's Sarcoma

high grade lesions in any bone; most often second decade, median age 14 occasional in adults; slight male predominance; rare in blacks; radiolucent permeative appearance, "onion skin like" with extension through cortex; xrays may be similar to ostemyelitis.

Statistical significance

how likely it is that an obtained result occurred by chance, expressed as a P value. If p <0.05 this means that there is less than 5 in 100 chance that we are rejecting a null hypothesis that is true, in other words saying that there is a difference or relationship between variables when there is not.

articular cartilage

hyaline cartilage that covers ends of bones in synovial joints

Qualitative study

in depth data collection. Usually involves interviews or surveys, small sample size. Ethnography, grounded theory.

Clinical management of osteomalacia

includes supplementation of Ca++, Vit D, and protein in the diet. If there is an associated intestinal disorder, it must be referred for treatment. Any proposed treatment also must take into consideration appropriate treatment for underlying conditions such as hyperthyroidism, intestinal disorders and CKD.

Acute clinical management of gout

includes weight bearing restriction until after the acute episode subsides. NSAIDs are the drug of choice for younger patients w no other health problems. Uricosurics are used for prevention of recurrent attacks and decrease serum uric acid by increasing renal excretion (e.g. PROBENECID). ALLOPURINOL is used for chronic treatment only. FEBUXOSTAT and biologic agents are in development (e.g. interleukin 1 inhibitors and pegloticase) and represent substantial therapeutic advances, particularly for severe, treatment-refractory gout, and pts w comorbidities or intolerance to other drugs.

Penetrating trauma

injury caused by an object that passes through the skin or other body tissues. High velocity: missiles/bullets; crush/stretch. Low velocity: stab wounds; shearing/stretching. High pressure: injection/explosion; tearing.

Ratio

interval plus absolute zero, eg wt/ht, does not exist in psychosocial realm

Vertebral dislocation

involves sprained or ruptured ligaments causing the vertebra to move, also called spondylolisthesis

A Pavlik harness

is used in an infant with DDH to place the hips in abduction and proper alignment

Research problem

issue you want to address

Genu valgus

knock knees. Joint laxity of the medial collateral ligaments. Common in 3-5 yo, improves by 7 yo. Normal angulation: girls: 7- 9 degrees valgus; boys: 4-6 degrees valgus

Osgood-Schlatter disease

known as osteochondritis of the tibial tubercle. It is a painful lump below the kneecap in children with growth spurts. It is more common in children who are active in sports, more common in boys, and often bilateral. It is usually self-limiting.

Load and Shift Test (Drawer Test)

laxity test of the shoulder. This test is used to determined how loose the shoulder ligaments are. There are several variations of this test in use. One of the most common involves having the pt lie flat on the back so that the center of the shoulder blade is on the edge of the bed. The provider holds the arm out at 90 d from the side to see how much movement there is in the shoulder joint in the anterior direction and the posterior direction. The distance the arm can be moved in this position can be scored for a rough indication of the shoulder's stability or instability.

Neurofibormatosis type 2

less common than type 1. Sometimes referred to as a Central NF or Vestublar NF. Classic of this type are bilateral tumors, acoustic neuromas of the 8th cranial nerve leading to deafness - changes in vision - cataracts - in the brain, meningiomas occur less than 10%. It is often not diagnosed until 20 years of age.

Incomplete fracture

line of fracture does not include the whole bone, usually non-displaced

Aims of surgical management of bone metasasis

local tumor control; durable stability; immediate reduction of pain; acceptable function of involved extremity as quickly as possible; prophylactic stabilization of impending pathologic fractures

Quasi-experimental

manipulation of independent variable, but does not have a control group, or dose not have random assignment

Condylar

pertaining to a rounded projection on a bone

Multivitamin should not be taken with methotrexate because:

methotrexate is a folic acid antagonist. Oral care is very important to continue, and methotrexate is teratogenic - be sure patient is taking precautions to not get pregnant and wait 8 weeks after stopping methotrexate to become pregnant.

Type II open fracture

moderate contamination, > 1 cm, moderate soft tissue injury

DDH (developmental dysplasia of the hip) risk factors

more common in a 1st born, breech birth, when parent or sibling had DDH; females have 6:1 ratio. DDH refers to range of hip dysplasias from shallow acetabulum to a subluexable hip, to a totally dislocated hip. Most cases are detectable at birth.

Prevention of ACL tears

more common in women (4-6x) possibly related to hormonal differences, lax ACL, narrow intercondylar notch, wider pelvis, increased q-angles, less strength, slower rxn times. Prevention includes strengthening of hamstrings (to help keep the tibia in place during landings from jumps and sudden stops), and increase the speed with which hamstrings react to ACL-stressing movement.

Components of the diagnosis of bone neoplasms

morphology; border of the lesion; done destruction; perisosteal reaction; matrix of the lesion; soft tissue mass

Neurofibromatosis type 1 VonRecklinghausen's

most common as compared to type 2. Associated with cafe-au-lait spots and fibromas. The central form has bilateral acoustic neuromas and no peripheral deficits. Diagnosis is made by the # and size of cafe-au-lait spots which increase during the first years. Skeletal abnormalities include thickening of the cortex of long bones - can result in bowing and scoliosis.

Elbow dislocation

mostly posterior - defined by position of olecranon relative to humerus. Posteriolateral MC and occur as the result of hyperextension of elbow from a fall on outstretched UE. posterior dislocations frequently cause avulsion fractures of medial epicondtyle secondary to traction pull of medial collateral ligment. with complete dislocation, UCL will rupture. plain x-ray. clinical signs: rapid swelling, severe pain at elbow and deformity

Nominal

naming: gender, blood type, type of RN staffing, marital status, etc

Athletic injury assessment: test structural integrity

nerves: sensory and motor; circulation: feel pulse in injure body part; MS: look for abnormal ROM &/or pain in and around both injured & surrounding joints: active ROM, passive ROM, resistive ROM; special tests

Myelodysplasia/Spina Bifida

neural tube defect resulting in vertebral and/or spinal cord malformation. Promote mobility, ADL's, diet, bowel bladder program; latex precautions; neuro status - shunt problems; insensate skin, water temp, braces; multiple orthopedic corrections.

Osteoblastic bone formation

new bone formed in response to cancer spread; abnormal growth which causes bone to be weak and deformed; more frequently seen in spread of prostate, bladder and stomach cancer

Rotator cuff injury

nighttime shoulder pain, shoulder weakness

Type I open fracture

no contamination, <1 cm size, minimal soft tissue injury

Non-experimental

no manipulation of independent variable, no control group or random assignment

Arthrogryposis

non progressive joint contractures that begin prenatally; flexion contractures at knees, elbows, fingers with muscle weakness around the joint. Etiology: unknown; 0.03% of general population. Dx: clinical exam xray, muscle biopsy.

Bisphosphonates

non-hormonal agents. Highly selective osteoclast inhibitor (stop breakdown). Indicated for tx & prevention of osteoporosis in men & women. BMD 2 standard deviations below normal for young adults also an indication. Potential s/e GI distress/disorder/esophageal & gastric ulcers. Caution in renal dysfunction. Osteonecrosis of the jaw. Drug holiday after 3-5 yrs. Safety: concerns about over-suppresion of bone turnover (concern of impaired fracture healing); atypical fragility fractures; osteonecrosis of the jaw (ONJ); a-fib?

Avascular necrosis of the hip

occurs with unstable hip d/t loss of blood supple to the femoral head. Etiology is unknown, but could be related to endocrine, trauma, mechanical, or systemic problems. Boys are affected 2-3 times more often, boys age 10-17, girls age 8-15. There is hip and knee pain of long duration. Pain may be dull and long-term with complaints of groin pain.

Risk factors for NSAID ulcers

older age. Prior h/o peptic ulcer or GI symptoms w NSAIDs. Concomitant use of prednisone. NSAID dose: more prostaglandin suppression = greater risk fo serious events. Disability level: the sicker the pt the higher the risk

Ordinal

ordering - putting subjects in order high to low. Can't say anything about magnitude of differences (low, medium, high)

Benign bone tumors

osteochondroma, enchondroma, and giant cell tumor

The second most common primary bone cancer:

osteosarcoma

shin splints (medial tibial stress syndrome)

pain along medial side of tibia, usually inflammation of periosteum, overuse injury from running on hard surfaces or on tiptoes. Can be a stress fracture. Return to practice/game: sidelined 1-2 weeks, must be pain free after prolonged run.

Pain w metastatic bone disease

pain is the most common symptoms associated with skeletal disease; location of lesions determine pain onset - weight-bearing activities typically present early whereas ribs or sternum (axial skeleton) may remain asymptomatic until later and experience pathologic fractures. Characteristics: constant; gradually progressive in intensity; exacerbation by change in position, movement, change in body posture. Pain w/o activity (lifting or walking) is particularly concerning in a cancer patient.

Cervical disk disease

pain radiating below elbow decreased cervical range of motion

Complications of bony metasases

pain; neurologic impairment; pathologic fracture; hypercalcemia of malignancy; anemia

Impact of metastatic bone disease

painful in area of spread; damaging to bone, weakens bone; puts patients at risk for broken bones; can impact people's ability to participate in daily activities; biggest concern of cancer patients are pain management and maintaining quality of life

Osteosarcoma

peak in second decade; M>F; 1/2 around knee, rare in wrist, handle, ankle, foot; variable, highly destructive, radiolucent mixed with radiodense areas; margins ill-defined

simple vertebral fracture

pedicle, facet, spinous process or transverse process fracture, usually stable

Tibial torsion

physiologic bowing of tibia; 2 degree intrauterine position; NORMAL for toddler; no tx unless present at 8 yr o; nursing: bring grandparent to appointment

Shoulder dislocation diagnostics

plain xrays; CT scans if complicated associated fx; MRI rarely - get more info from arthroscopy; MRI w arthrogram - capsular or labral injury w rotator cuff tear; examination under general anesthetic; supine load shift test w arm at 80 degree compared w normal shoulder; 1+ ball to rim; 2+ ball riding over irm w spontaneous reduction; 3+ ball stays dislocated; arthroscopy

Diagnostic studies for tumors/cancer

plain xrays; CT scans; Technetium 99 pyrophosphate bone scans - most sensitive in detecting skeletal metastases; MRI w gadolinium contrast; Indium 111 scans; Thallium scans; arteriography

This shoulder dislocation is seen most often with seizure activity:

posterior shoulder dislocation, not as common.

Penetrating fracture

projectile penetrates bone (i.e. gunshot)

Skeletal traction

pull exerted directly on the skeletal system by attaching wires, pins, or tongs into or through a bone. Halo vest; Steinman pin or Kirschner wire w balanced suspension (Thomas splint w Pearson attachment) or BB (Bohler Braun) frame

Groin strain

pull of groin area (depression that lies between the thigh and the abdominal region); may include hip adductors, hip flexors, or abdominal. Mechanism of action: overextension of the groin. Return to practice/game: sidelined 2-3 weeks; run, run figure 8 around cones, all activities must be pain free.

Research question

question you want answered

Interval

rank-ordered but w equal distance between ranks (we pretend Likert scales do this); no absolute zero; e.g. pain scores

Multiple Myeloma

rare before 5th decade of life; M<F; multifocal bone pain, anemia; test UPEP & SPEP to demonstrate monoclonal protein (M spike); ribs, vertebrae, skull and pelvis; punched out lesions w/o surrounding bony reaction; most common malignant bone tumor.

Purpose of traction

reduce, realign, and promote healing of fractured bones; rest joints; treat dislocations and subluxations; decrease muscle spasms; immobilization; reduce deformities; treat deformities; expand joint space

epiphyseal line

remnant of the epiphyseal plate, seen in adult bones

Shoulder tendinitis treament

rest; oral meds - NSAIDs; cuff strengthening; cortisone injection; PT; surgery - biceps tendesis/cuff repair

Femoral anteversion description

rotation of the femur along it's long axis. Generally noted during preschool and resolves by skeletal maturity. This problem is considered excessive if it's beyond 50% or causes functional problems. "W" sitting should be discouraged in these children.

In the sitting position, areas especially susceptible to pressure injuries include the patient's:

scapulae, ischial tuberosities and calcaneous

Serratus anterior or trapezius injury findings

scapular winging, trauma, recent viral illness, dysfunction

In most cases of musculoskeletal tumors, many patients present with:

sever pain following a trauma that does not resolve with conservative measures. It is helpful to note the duration and onset of patient's symptoms. Pain may be present at night or when at rest. Other symptoms of bone cancer can include bone pain, swelling and tenderness near the affected area, weakened bone leading to fracture, fatigue and unintended weight loss. Key factors during the PE include whether or not they have a palpable mass, enlarged lymph nodes, or enlargement of the liver. Patient may avoid weight bearing on the affected limb due to pain. We would also expect to see liver or lymphatic involvement.

Crush fracture

severe soft tissue injury w transverse or comminuted fracture

Diaphyseal

shaft of a long bone

Iodine is an agent found in many radiography dyes and antimicrobial skin preparations. Patients allergic to iodine may mention an allergy to:

shellfish

Stimson's technique

shoulder relocation technique: appropriate wt (e.g. 5 lbs) are taped to the wrist of the dislocated shoulder which hangs free over the edge of the table. If meds are used, the patient should be monitored as it may take 15 or 20 minutes for the reduction to occur.

Matsen's technique

shoulder relocation technique: the pt lies on the back with a sheet around the chest and also around the assistant's waist for countertraction. The surgeon stands on the side of the dislocated shoulder near the pt's waist w the elbow of the dislocated shoulder bent to 90 d. A second sheet, tied loosely around the surgeon's waist and looped over the pt's forearm, provides traction while the surgeon leans back against the sheet while grasping the forearm. Steady traction along the axis of the arm usually causes reduction.

Type III-C open fracture

significant contamination, > 1 cm, any degree of soft tissue damage with arterial injury

Type III open fracture

significant contamination, > 1 cm, extensive soft tissue injury possibly needing amputation

Type III-B open fracture

significant contamination, >1 cm, bony exposure, periosteum stripped, severe injury with tissue loss

Type III-A open fracture

significant contamination, >1 cm, severe soft tissue injury but good tissue coverage over the fracture

Autograft

skin graft from a person's own body

Allograft

skin graft from another person or a cadaver

Knee injury exam: vascular exam

skin temp/color; capillary refill; popliteal pulse; dorsalis pedal pulse; posterior tibialis pulse

Osteomalacia

softening of the bones, typically through a deficiency of vitamin D or calcium. Soft bones in adults, known as rickets in children. Growth plates fail to mineralize yet thicken and become irregular. Bone effects: decreased Ca++ and phosphate needed for bone formation; decreased Vit D; decreased intestinal absorption of Ca++; decreased bone strength, formation, resorption. Major s/s are muscle weakness and achy bones.

Variables

something that varies, is not constant: independent variable: a presumed cause; dependent variable: the presumed effect or outcome; confounding variable: one that has an effect on the dependent variable

Clinical signs of malignant hyperthermia

specific signs: muscle rigidity; increased CO2 production; rhabdomyolysis; marker temperature elevation. Non-specific signs: tachycardia; tachypnea; acidosis (respiratory/metabolic); hyperkalemia

Most common sites of bony metastases

spine; pelvis; ribs; upper arm to humerus; skull; long bones in legs

Retrospective data

starts w dependent variable and looks for cause

Prospective data

starts with independent variable and looks for effect

Traction fracture

strong pull at the insertion of a tendon or ligament causing an avulsion

Open reduction Internal Fixation (ORIF)

surgical alignment of fractured bones using screws, pins, wires, or nails to maintain bone alignment. Used for displaced fractures, nonunion, reattachment of amputation, significant tendon or ligament avulsion fractures, inability to do closed reduction, or pathologic fracture.

Signs of inflammation

swelling, warmth, erythema, loss of function, tenderness

Treatment for Stingers/Burners

symptoms less than 3 weeks: return to play allowed if no neck symptoms, full ROM, full strength and normal neuro exam. Symptoms greater than 3 weeks or multiple episodes: require radiographic evaluation and consideration of restriction from play if 3 episodes.

Athletic injury assessment: Palpation

tell athlete it may hurt and why; begin gently & gradually increase pressure; correlate athletes reaction to structures palpated; start away from suspected injury and gradually work toward it.

Apophyseal

tension point at growth plate

All people with Ankylosing Spondylitis have:

the HLA-B26 gene. This is a type of arthritis that affects the spine, including the sacroiliac joints, intervertebral spaces, costovertebral articulations, shoulder and hip joint and where tendons and ligaments attach to bone, mainly in the spine but sometimes along the back of the heel. The vertebrae fuse together - making the spine rigid. While the precise etiology is unknown, there is a strong family link - the HLA-B27 gene. Even with that, 75% of people carrying the gene will NOT develop the disease. It is more often seen in males, generally between the ages of 25-34 years old. Non known to lead to paralysis. The ligaments and tendons become inflamed (tendinitis) and cause stiffness and pain. The synovial tissues becomes inflamed. Pannus forms the spine and the peripheral joints such as hip and shoulder are affected. The process creates dense fibrous scars

Metaphyseal

the flared portion of the bone at the ends of the shaft

Myelomeningocele (spina bifida)

the lack of fusion without neuro defects; cystica is a fusion defect with hernial meninges; myelomeningocele is the lack of fusion of vertebral arches dysplasia of spinal cord and membranes. There is partial or complete paralysis and sensory loss below the affected area, usually lumbar defect. 90% have hydrocephalus. The cause is multifactoral-genetic, decreased folic acid, or exposure to valporic acid during pregnancy. Symptoms have wide range from associated neuro symptoms, orthopedic spinal problems, urinary and bowel complications, or infection. Nursing care and parental care focus on any and all of the range of symptoms and systems. LATEX ALLERGY is common in these children. Parents need to be taught symptoms of shunt malfunction: altered LOC, blurred vision, and increased irritability.

Osteochondromas

the most common benign tumors of the bones and take the form of cartilage-capped bony projections or outgrowth on the surface of bones, characterized as overgrowth in any bone where cartilage forms bone.

Mode

the most frequent response

Adrenal cortex

the outer portion of the adrenal glands. Responsible for glucocorticoids, minerals. ACTH to cholesterol and steroidgenesis. Glucocorticoids decrease the bone matrix growth. Mineral corticoids contribute to electrolyte balance.

A drawback of close-ended questions is:

the risk of failing to include key responses. Such omissions can lead to inadequate understanding of the phenomenon or to outright bias if respondents choose an alternative that misrepresents their position. Open-ended questions require more time-consuming analysis and carry greater risk for unanswered questions, but they do allow participants to elaborate in their answers.

Median

the value below which 50% of the cases fall

Quadriceps Contusion

thigh bruise; most common of all thigh injuries. Often called 'dead leg' or 'charley horse'. Mechanism: blunt force trauma to the muscle. Tx: intense PT for motion. Complications: myositis osificans. Return to practice/game: run, run w change of motion, jumping, all activities must be pain free. Sidelined 2-3 weeks.

Lesion

tissue/organ abnormality; hematoma (blood clot); cyst; aneurysm; inflammation

Rotator cuff tear

traumatic rip of one or more of the muscles or tendons within the rotator cuff of the shoulder. Often with suprascapular nerve entrapment. Findings: supraspinatus/infraspinatus wasting.

Osteolytic bone destruction

tumor completely destroys bone at site of spread; most common in cancers spread to bone from lung, thyroid, kidney and colon

Rotational fracture

twisting stress causes spiral type of fracture

External fixation indications

used for acute or angulated fxs, limb length discrepancies and non-union fxs. Beneficial when there is significant soft tissue deformity or open wounds. It can also assist w the correction of deformity as a result of a fx that healed in an anatomically poor position. Contraindications include a pt w compromised immune system; a non-compliant pt who would not be able to ensure proper wire and pin care; pre-existing internal fixation that prohibits wire or pin placement; bone pathology precluding pin fixation.

Cholchicine

used to decrease inflammation by reducing the migration of leukocytes to synovial fluid.

Quantitative study

usually numeric data, statistical analysis, larger sample size. Answers questions about whether one group is different from another, can describe a sample numerically.

Acetabular fracture

usually result from a fall or impact from MVA

sacrococcygeal fracture

usually result from fall landing flat on sacrococcygeal area

Physical findings of DDH

vary w the severity of the pathology and the age of the child. Don't get caught thinking of all hips as 'dislocated': asymmetric thigh fold; limited abduction; positive Galeazzi; pistoning; Trendelenburg gait. The hip will be dislocated in the resting position. Dislocatable with the Barlow maneuver. May stabilize spontaneously or with treatment. May convert to dislocated without tx. May reduce with the Ortolani maneuver; this will be lost, usually by 6-8 weeks of age. Will not resolve w/o tx: Pavlik harness if criteria met.

Nuerofibromatosis type 3 Swannomatosis

very rare. Cranial-spinal and peripheral nerves develop tumors - chronic pain, numbness, tingling and muscular weakness result - this can be limited to an arm, leg, or the spine. Most have back pain and scoliosis. The peripheral cell tumors predispose them to cancer. Tx involves management of symptoms and complications. This includes surgery to remove or debulk tumors, radiation to reduce the size of inoperable tumors, and auditory brain stem implants for those with NF2 who have parts of the auditory nerve intact. Hearing aids are generally not helpful.

Neurofibromatosis

von Recklinghausen's disease. Growth of multiple tumors from the nerve sheath. Progressive. Tumors in central and peripheral nervous system. 1:400,000 births M=F, 4 stages. Dx: Cafe au lait spots >6; Lisch nodules in iris

Exercise for healthy bones

weight bearing; high impact; low impact; resistance & strengthening; non-impact; balance; posture; function

Hypothesis

what you think will happen

intra-articular

within a joint

Multidirectional instability

• GENERALLY LAX: Physiologically lax connective tissue • Traumatic injury with capsule/ligament tears • Repetitive microtrauma - Ligaments stressed with repetitive use and hypermobility - May cause subluxation, dislocation or RTC tendonitis - Long term: bone spurs, tendon rupture, capsular restrictions


Conjuntos de estudio relacionados

Chapter 39: Oxygenation and Perfusion

View Set

Hosea 10 - Flashcard MC questions - Ted Hildebrandt

View Set

Second Half of Bus Org Quiz Questions

View Set

Chapter 51, Care of the Patient with a Reproductive Disorder

View Set

Pathophysiology: GI Disorders Questions

View Set

Fat-Soluble Vitamins A,D,E,K in detail

View Set

MES7: Geology and Geologic Hazards

View Set

GEOL 1005 CH. 9, 10, 12, 13, 14, 15

View Set

Implementation and Evaluation of Interventions Related to Sleep

View Set